You are on page 1of 74

TAMPUS, MARY GRACE G.

CONSTITUTIONAL LAW 1

Manila Prince Hotel vs. GSIS (G.R.No. 122156, February 3, 1997)

FACTS:

GSIS decided to sell through public bidding, 30%-51% of the issued and outstanding shares of Manila Prince Hotel.
Hence, in a close bidding, only 2 bidders participated:

- Manila Prince Hotel Corporation offering to buy 51% of MPH at P41.58 per share
-Renong Berhad, a Malaysian firm, also offering to buy 51% of MPH but at P44 per share

As to the rules in bidding, it was stipulated that: 1) The Highest bidder must comply to the conditions or else will lose
the right to purchase the block of shares and instead will be given to other qualified bidders 2) The Highest bidder
shall only be declared the winning bidder after execution of necessary contracts with GSIS.

Pending the declaration of Renong Berhad as the winning bidder and execution of necessary contracts, Manila
Prince Hotel Corporation sent a letter to GSIS matching the bid price of P44 per share tendered by Renong Berhad
which MPHC readily sent a tendered check, however, was refused by GSIS.

Anxious that GSIS has disregarded the tendering of check to match the bid and that sale may be hastened, MPHC
filed a complaint for prohibition and mandamus.

MPH alleges that under Sec.10, Art 12 of the 1987 Constitution, Manila Hotel has been identified with the Filipino
nation and has practically became a historical monument which reflects the vibrancy of PH culture and heritage. It is
a proud legacy of an earlier generation of Filipinos who believed in the nobility and sacredness of independence and
its power and capacity to release the full potential of Filipino people, and that to all intents and purposes, it has
become a part of national patrimony. Therefore, if Manila Hotel is part of national patrimony and its business also a
part of national economy then MPH should be preferred after it has matched the bid offer of the Malaysian Firm. For
the bidding rules mandate that if for any reason, the Highest Bidder cannot be awarded of the block of shares, GSIS
may offer it to other qualified bidders that have validly submitted bids provided that these qualified bidders are willing
to match the highest bid in terms of price per share.

GSIS however responded that the allegations of MPH are misplaced. Sec.10, Art 12 of the 1987 Constitution is
merely a statement of principle and policy since it is not a self-executing provision and requires implementing
legislations for the said provision to operate. GSIS further argues that suppose the provision is self-executing,
arguments of MPH will still not prosper considering that Manila Hotel does not fall under the term national patrimony
as it only refers to lands of the public domain, waters, minerals, coal, petroleum and other mineral oils, all forces of
potential energy, fisheries, forests or timber, wildlife, flora and fauna and all marine wealth in its territorial sea and
exclusive marine zone as cited in the 1st and 2nd paragraphs of Section 2, Article 12 of the 1987 Constitution. The
events that have transpired in the hotel making it historic does not make the hotel fall under the patrimony of the
nation.

ISSUE:

1. WON Article 12, Sec.10, par.2 of the 1987 Constitution is a self-executing provision
TAMPUS, MARY GRACE G. CONSTITUTIONAL LAW 1

2. WON Manila Hotel forms part of national patrimony hence granting MPH right of preference over shares of
stock

HELD:

1. Yes, the said provision is a self-executing one.


A constitution is a system of fundamental laws for the governance and administration of a nation. It is
supreme, imperious, absolute, and unalterable except by the authority from which it emanates. It has been
defined as the fundamental and paramount law of the nation. It prescribes the permanent framework of a
system of government, assigns to the different departments their respective powers and duties, and
establishes certain fixed principles on which government is founded.
A provision which lays down a general principle, such as those found in Article 2 of the 1987 Constitution is
usually not self-executing but a provision which is complete in itself and becomes operative without the aid
of supplementary or enabling legislation, or that which supplies sufficient rule by means of which the right it
grants may be enjoyed or protected, is self-executing. Thus a constitutional provision is self-executing if the
nature and extent of the right conferred and liability imposed are fixed by the Constitution itself, so that they
can be determined by an examination and construction of its terms, and there is no language indicating that
the subject is referred to the legislature for action. Hence, unless, it is expressly provided that a legislative
act is necessary to enforce a constitutional mandate, presumption now is that all provisions of constitution
are self-executing. If the constitutional provisions are treated as requiring legislation instead of self-
executing, legislature would have the power to ignore and practically nullify mandate of fundamental law.
Quite apparently, Sec.10, 2nd par. of Article 12 is couched in such a way as not to make it appear that it is
non-self-executing but simply for purposes of style. It is a mandatory, positive command which is complete
in itself and which needs no further guidelines or implementing laws or rules for its enforcement. From its
very words the provision does not require any legislation to put it in operation. It is per se judicially
enforceable.

2. Yes, the Manila Hotel forms part of the patrimony.


In its plain and ordinary meaning, patrimony pertains to heritage. When the Constitution speaks of national
patrimony, it refers not only to the natural resources but also to the cultural heritage of the Filipinos. Manila
Hotel has become a landmark a living testimony of PH heritage. While it was restrictively an American
hotel when it first opened, it immediately evolved to be truly Filipino as it has since become the venue of
various significant events which have shaped Philippine history. Considering that Manila Hotel forms part of
the national patrimony it shall be rightful then to give MPH the preference to purchase the shares of stock as
per mandated in Sec.10, 2nd par. of Article 12 of 1987 PH Constitution considering that our Constitution
mandates that in the grant of rights, privileges, and concessions covering national economy and patrimony,
State shall give preference to qualified Filipinos. And when our Constitution declares that a right exists in
certain specified circumstances an action may be maintained to enforce such right notwithstanding the
absence of any legislation on the subject; consequently if there is no statute especially enacted to enforce
TAMPUS, MARY GRACE G. CONSTITUTIONAL LAW 1

such constitutional right such right enforces itself by its own inherent potency and puissance, and from
which all legislations must take their bearings.
To insist on selling the Manila Hotel to foreigners when there is a Filipino group willing to match the bid of
foreign group is to insist that government be treated as any other ordinary market player, and bound by its
mistakes or gross errors of judgment, regardless of the consequence to Filipino people. Since MPH has
already matched the bid price tendered by Renong Berhad pursuant to the bidding rules, GSIS is left with no
alternative but to award MPH the block of shares and execute the necessary agreements and documents to
effect the sale in accordance not only with the bidding guidelines and procedures but with the Constitution
as well. The refusal of GSIS to execute the corresponding documents with MPH as provided in bidding rules
after the latter has matched the bid of Malaysian firm clearly constitutes grave abuse of discretion.
TAMPUS, MARY GRACE G. CONSTITUTIONAL LAW 1

Oposa vs. Factoran (G.R.No.101083, July 10, 1993)

FACTS:

The complaint was instituted as taxpayers class suit and alleges that the plaintiffs (Oposa) are all citizens of the PH,
taxpayers and entitled to the full benefit, use and enjoyment of natural resource treasure that is the countrys virgin
tropical rainforests. The same was filed for themselves and others who are equally concerned about the preservation
of said resource but are so numerous that it is impracticable to bring them all before the Court. The minors further
asservated that they represent their generation as well as generations yet unborn to order the cancellation of all
existing timber license agreements in the country; cease and desist from receiving, accepting, processing, renewing,
or approving of new TLAs considering the detrimental consequences of continued deforestation anchoring such
arguments under Arts. 19,20 and 21 of the Civil Code, Sec.4 of EO 192 creating DENR and Article 2 of the 1987
Constitution recognizing the right of people to a balanced and healthful ecology.

Factoran however avers that the complaint is nothing but vague and nebulous allegations concerning an
environmental right and that it does not constitute as a cause of action. Furthermore, the question of whether logging
should be permitted in the country is a political question which should be properly addressed to the executive or
legislative branches of the Government.

ISSUE/S:

1. WON the petitioners have a cause of action


2. WON the complaint raises a political question or a justiciable question

HELD:

1. Yes, they have a cause of action.


The right of the petitioners to a balanced and healthful ecology is as clear as DENRs duty under its
mandate and by virtue of its powers and functions under EO 192 to protect and advance said right. A denial
or violation of that right by the other who has the correlative duty or obligation to respect or protect the same
gives rise to a cause of action. While the right to a balanced and healthful ecology is to be found under
Declaration of Principles and State Policies it does not follow that it is less important than any of the civil and
political rights enumerated in the Constitution. As a matter of fact, these basic rights need not even be
TAMPUS, MARY GRACE G. CONSTITUTIONAL LAW 1

written in the Constitution for they are assumed to exist from the inception of humankind. If they are now
explicitly mentioned in the fundamental charter, it is because of the well-founded fear of its framers that
unless the rights to a balanced and healthful ecology and to health are mandated as state policies by the
Constitution itself, thereby highlighting their continuing importance and imposing upon the state a solemn
obligation to preserve the first and protect and advance the second, the day would not be too far when all
else would be lost not only for the present generation, but also for those to come generations which stand
to inherit nothing but parched earth incapable of sustaining life. The right to a balanced and healthful
ecology carries with it the correlative duty to refrain from impairing the environment.
2. It must, nonetheless, be emphasized that the political question doctrine is no longer the insurmountable
obstacle to the exercise of judicial power or the impenetrable shield that protects executive and legislative
actions from judicial inquiry or review. The second paragraph of section 1, Article VIII of the Constitution
states that: "Judicial power includes the duty of the courts of justice to settle actual controversies involving
rights which are legally demandable and enforceable, and to determine whether or not there has been a
grave abuse of discretion amounting to lack or excess of jurisdiction on the part of any branch or
instrumentality of the Government." Commenting on this provision in his book, Philippine Political Law, Mr.
Justice Isagani A. Cruz, a distinguished member of this Court, says: "The first part of the authority
represents the traditional concept of judicial power, involving the settlement of conflicting rights as conferred
by law. The second part of the authority represents a broadening of judicial power to enable the courts of
justice to review what was before forbidden territory, to wit, the discretion of the political departments of the
government. As worded, the new provision vests in the judiciary, and particularly the Supreme Court, the
power to rule upon even the wisdom of the decisions of the executive and the legislature and to declare their
acts invalid for lack or excess of jurisdiction because tainted with grave abuse of discretion.
TAMPUS, MARY GRACE G. CONSTITUTIONAL LAW 1

FRANCISCO et al vs. HOUSE SPEAKER (G.R.No.160261, November 10, 2003)

FACTS:

Former President Estrada filed an impeachment complaint against Chief Justice Hilario Davide, Jr. and seven
Associate Justices for culpable violation of the Constitution, betrayal of public trust and other high crimes. The
complaint was endorsed and then was referred to the House Committee on Justice in accordance with Section 3(2)
of article 11 of the Constitution:

A verified complaint for impeachment may be filed by any Member of the House of Representatives or by
any citizen upon a resolution of endorsement by any Member thereof, which shall be included in the Order
of Business within ten session days, and referred to the proper Committee within three session days
thereafter. The Committee, after hearing, and by a majority vote of all its Members, shall submit its report to
the House within sixty session days from such referral, together with the corresponding resolution. The
resolution shall be calendared for consideration by the House within ten session days from receipt thereof.

A day after the House Committee on Justice voted to dismiss it, 2nd impeachment complaint was accompanied by
Resolution of Endorsement/Impeachment signed by at least 1/3 of all members of the House of Representatives.
Prompting the petitions filed by members of the Bar, the House of Reps and private individuals to stop the illegal
spending of public funds for the impeachment proceedings against the Chief Justice asserting that the filing of the 2nd
impeachment complaint is barred under Article 11, Sec.3 (5) of the 1987 Constitution stating that, no impeachment
proceeding shall be initiated against the same official more than once within a period of 1 year.

Respondent House of Representatives however asserted that the Court has no jurisdiction to hear, much less
prohibit or enjoin House of Representatives considering it is an independent and co-equal branch of the Government
under the Constitution.

ISSUE/S:

1. WON the power of Judicial Review extends to those arising from impeachment proceedings
2. WON the filing of the 2nd impeachment complaint violates the 1 year ban as to impeachment proceedings
provided in the Constitution.
TAMPUS, MARY GRACE G. CONSTITUTIONAL LAW 1

HELD:

1. Yes. Supreme Court held that the 2nd impeachment complaint filed against Chief Justice Hilario Davide, Jr.
was unconstitutional.

Judicial Power includes the duty of the Courts of Justice to settle actual controversies involving rights which are
legally demandable and enforceable, and to determine whether or not there has been grave abuse of discretion
amounting to lack or excess of jurisdiction on the part of any branch or instrumentality of the Government.

Judicial review is essential for the maintenance and enforcement of the separation of powers and the balancing of
powers among the three great departments of government through the definition and maintenance of the boundaries
of authority and control between them.

There is indeed a plethora of cases in which this Court exercised the power of judicial review over congressional
action. Thus, inSantiago v. Guingona, Jr., this Court ruled that it is well within the power and jurisdiction of the Court
to inquire whether the Senate or its officials committed a violation of the Constitution or grave abuse of discretion in
the exercise of their functions and prerogatives. Finally, there exists no constitutional basis for the contention that the
exercise of judicial review over impeachment proceedings would upset the system of checks and balances. Verily,
the Constitution is to be interpreted as a whole and "one section is not to be allowed to defeat another."

The exercise of judicial restraint over justiciable issues is not an option before this Court. Adjudication may not be
declined, because this Court is not legally disqualified. Nor can jurisdiction be renounced as there is no other tribunal
to which the controversy may be referred." Otherwise, this Court would be shirking from its duty vested under Art. VIII,
Sec. 1(2) of the Constitution. More than being clothed with authority thus, this Court is duty-bound to take cognizance
of the instant petitions. In the august words of amicus curiaeFather Bernas, "jurisdiction is not just a power; it is a
solemn duty which may not be renounced. To renounce it, even if it is vexatious, would be a dereliction of duty."

2. To determine the merits of the issues raised in the instant petitions, this Court must necessarily turn to the
Constitution itself which employs the well-settled principles of constitutional construction.

First, verba legis, that is, wherever possible, the words used in the Constitution must be given their ordinary meaning
except where technical terms are employed.

Second, where there is ambiguity, ratio legis est anima. The words of the Constitution should be interpreted in
accordance with the intent of its framers.

Finally, ut magis valeat quam pereat. The Constitution is to be interpreted as a whole.

If, however, the plain meaning of the word is not found to be clear, resort to other aids is available.

Father Bernas concludes that when Section 3 (5) says, "No impeachment proceeding shall be initiated against the
same official more than once within a period of one year," it means that no second verified complaint may be
accepted and referred to the Committee on Justice for action. By his explanation, this interpretation is founded on the
common understanding of the meaning of "to initiate" which means to begin. He reminds that the Constitution is
ratified by the people, both ordinary and sophisticated, as they understand it; and that ordinary people read ordinary
TAMPUS, MARY GRACE G. CONSTITUTIONAL LAW 1

meaning into ordinary words and not abstruse meaning, they ratify words as they understand it and not as
sophisticated lawyers confuse it. To the argument that only the House of Representatives as a body can initiate
impeachment proceedings because Section 3 (1) says "The House of Representatives shall have the exclusive
power to initiate all cases of impeachment," this is a misreading of said provision and is contrary to the principle of
reddendo singula singulis by equating "impeachment cases" with "impeachment proceeding."

From the records of the Constitutional Commission, to the amicus curiae briefs of two former Constitutional
Commissioners, it is without a doubt that the term "to initiate" refers to the filing of the impeachment complaint
coupled with Congress' taking initial action of said complaint. Having concluded that the initiation takes place by the
act of filing and referral or endorsement of the impeachment complaint to the House Committee on Justice or, by the
filing by at least one-third of the members of the House of Representatives with the Secretary General of the House,
the meaning of Section 3 (5) of Article XI becomes clear. Once an impeachment complaint has been initiated,
another impeachment complaint may not be filed against the same official within a one year period.

In fine, considering that the first impeachment complaint, was filed by former President Estrada against Chief Justice
Hilario G. Davide, Jr., along with seven associate justices of this Court, on June 2, 2003 and referred to the House
Committee on Justice on August 5, 2003, the second impeachment complaint filed by Representatives Gilberto C.
Teodoro, Jr. and Felix William Fuentebella against the Chief Justice on October 23, 2003 violates the constitutional
prohibition against the initiation of impeachment proceedings against the same impeachable officer within a one-year
period.

WHEREFORE, Sections 16 and 17 of Rule V of the Rules of Procedure in Impeachment Proceedings which
were approved by the House of Representatives on November 28, 2001 are unconstitutional.
TAMPUS, MARY GRACE G. CONSTITUTIONAL LAW 1

Santiago v. Guingona (G.R.No.134577, Nov. 18, 1988)

FACTS

The Senate of the Philippines convened on July 27, 1998 for the first regular session of the eleventh Congress. On
the agenda for the day was the election of officers. Nominated by Sen. Blas F. Ople to the position of Senate
President was Sen. Marcelo B. Fernan. Sen. Francisco S. Tatad was also nominated to the same position by Sen.
Miriam Defensor Santiago. By a vote of 20 to 2, Senator Fernan was declared the duly elected President of the
Senate. The following were likewise elected: Senator Ople as president pro tempore, and Sen. Franklin M. Drilon as
majority leader.

Senator Tatad thereafter manifested that, with the agreement of Senator Santiago, allegedly the only other member
of the minority, he was assuming the position of minority leader. He explained that those who had voted for Senator
Fernan comprised the "majority," while only those who had voted for him, the losing nominee, belonged to the
"minority." the majority leader informed the body that he was in receipt of a letter signed by the seven Lakas-NUCD-
UMDP senators, stating that they had elected Senator Guingona as the minority leader. By virtue thereof, the Senate
President formally recognized Senator Guingona as the minority leader of the Senate. The following day, Senators
Santiago and Tatad filed before this Court the subject petition for quo warranto, alleging in the main that Senator
Guingona had been usurping, unlawfully holding and exercising the position of Senate minority leader, a position that,
according to them, rightfully belonged to Senator Tatad.

Petitioners (Santiago and Tatad) allege that the Court has jurisdiction to settle the issue as to who is the lawful
Senate minority leader. Respondents (Guingona) on the other hand, contend that the issue on deciding who the
lawful Senate minority leader is an internal matter pertaining exclusively to the domain of the legislature hence, Court
cannot exercise jurisdiction as by doing so, it will transgress the principle of separation of powers.

ISSUE:

1. WON the Court has jurisdiction over the petition, WON the issue as to who shall be the Senate minority
leader constitutes a justiciable question
TAMPUS, MARY GRACE G. CONSTITUTIONAL LAW 1

2. WON the election of Senator Guingona as Senate minority leader constituted as violation of the Constitution

HELD:

1. Yes, the Court has jurisdiction.


The early case Avelino v. Cuenco cautiously tackled the scope of the Court's power of judicial review; that is,
questions involving an interpretation or application of a provision of the Constitution or the law, including the
rules of either house of Congress. Within this scope falls the jurisdiction of the Court over questions on the
validity of legislative or executive acts that are political in nature, whenever the tribunal "finds constitutionally
imposed limits on powers or functions conferred upon political bodies."

In Taada v. Cuenco, this Court endeavored to define political question. And we said that "it refers to 'those
questions which, underthe Constitution, are to be decided by the people in their sovereign capacity, or in
regard to which full discretionary authority has been delegated to the legislative or executive branch of the
government.' It is concerned with issues dependent upon the wisdom, not [the] legality, of a particular
measure."

The Court rules that the validity of the selection of members of the Senate Electoral Tribunal by the senators
was not a political question. The choice of these members did not depend on the Senate's "full discretionary
authority," but was subject to mandatory constitutional limitations. 20 Thus, the Court held that not only was
it clearly within its jurisdiction to pass upon the validity of the selection proceedings, but it was also its duty
to consider and determine the issue.

2. No, it does not constitute as violation of the Constitution.


We believe, however, that the interpretation proposed by petitioners finds no clear support from the
Constitution, the laws, the Rules of the Senate or even from practices of the Upper House. The term
"majority" has been judicially defined a number of times. When referring to a certain number out of a total or
aggregate, it simply "means the number greater than half or more than half of any total." The plain and
unambiguous words of the subject constitutional clause simply mean that the Senate President must obtain
the votes of more than one half of all the senators. Not by any construal does it thereby delineate who
comprise the "majority", much less the "minority," in the said body. And there is no showing that the framers
of our Constitution had in mind other than the usual meanings of these terms.

In effect, while the Constitution mandates that the President of the Senate must be elected by a number
constituting more than one half of all the members thereof, it does not provide that the members who will not
vote for him shall ipso facto constitute the "minority", who could thereby elect the minority leader. Verily, no
law or regulation states that the defeated candidate shall automatically become the minority leader.

While the Constitution is explicit on the manner of electing a Senate President and a House Speaker, it is,
however, dead silent on the manner of selecting the other officers in both chambers of Congress. All that the
Charter says is that "[e]ach House shall choose such other officers as it may deem necessary." 43 To our
mind, the method of choosing who will be such other officers is merely a derivative of the exercise of the
TAMPUS, MARY GRACE G. CONSTITUTIONAL LAW 1

prerogative conferred by the aforequoted constitutional provision. Therefore, such method must be
prescribed by the Senate itself, not by this Court.

In view of the foregoing, Congress verily has the power and prerogative to provide for such officers as it may
deem. And it is certainly within its own jurisdiction and discretion to prescribe the parameters for the
exercise of this prerogative. This Court has no authority to interfere and unilaterally intrude into that
exclusive realm, without running afoul of constitutional principles that it is bound to protect and uphold the
very duty that justifies the Court's being. Constitutional respect and a becoming regard for the sovereign
acts of a coequal branch prevents this Court from prying into the internal workings of the Senate. To repeat,
this Court will be neither a tyrant nor a wimp; rather, it will remain steadfast and judicious in upholding the
rule and majesty of the law.

To accede, then, to the interpretation of petitioners would practically amount to judicial legislation, a clear
breach of the constitutional doctrine of separation of powers. If for this argument alone, the petition would
easily fail.

While no provision of the Constitution or the laws or the rules and even the practice of the Senate was
violated, and while the judiciary is without power to decide matters over which full discretionary authority has
been lodged in the legislative department, this Court may still inquire whether an act of Congress or its
officials has been made with grave abuse of discretion. This is the plain implication of Section 1, Article VIII
of the Constitution, which expressly confers upon the judiciary the power and the duty not only "to settle
actual controversies involving rights which are legally demandable and enforceable," but likewise "to
determine whether or not there has been a grave abuse of discretion amounting to lack or excess of
jurisdiction on the part of any branch or instrumentality of the Government."

Absent any clear-cut guideline, in no way can it be said that illegality or irregularity tainted Respondent
Guingona's assumption and exercise of the powers of the office of Senate minority leader. Furthermore, no
grave abuse of discretion has been shown to characterize any of his specific acts as minority leader.
TAMPUS, MARY GRACE G. CONSTITUTIONAL LAW 1

Javellana v. Executive Secretary (G.R.No.36142, March 31, 1973)

FACTS

In 1973, Marcos ordered the immediate implementation of the new 1973 Constitution. Javellana, a Filipino and a
registered voter sought to enjoin the Exec Sec and other cabinet secretaries from implementing the said constitution.
Javellana averred that the said constitution is void because the same was initiated by the president. He argued that
the President is w/o power to proclaim the ratification by the Filipino people of the proposed constitution. Further, the
election held to ratify such constitution is not a free election there being intimidation and fraud.

ISSUE: Whether or not the SC must give due course to the petition.

HELD: The SC ruled that they cannot rule upon the case at bar. Majority of the SC justices expressed the view that
they were concluded by the ascertainment made by the president of the Philippines, in the exercise of his political
prerogatives. Further, there being no competent evidence to show such fraud and intimidation during the election, it
is to be assumed that the people had acquiesced in or accepted the 1973 Constitution. The question of the validity of
the 1973 Constitution is a political question which was left to the people in their sovereign capacity to answer. Their
ratification of the same had shown such acquiescence.
TAMPUS, MARY GRACE G. CONSTITUTIONAL LAW 1

Bondoc v. Pineda (201 SCRA 792)

FACTS

Emigdio Bondoc and Marciano Pineda were rivals for a Congressional seat in the 4thDistrict of Pampanga. Pineda
was a member of the Laban ng Demokratikong Pilipino (LDP). While Bondoc was a member of the Nacionalista Party
(NP). Pineda won in that election. However, Bondoc contested the result in the HRET (House of Representatives
Electoral Tribunal). Bondoc won in the protest and he was subsequently declared as the winner by the HRET.

Meanwhile, one member of the HRET, Congressman Juanito Camasura, Jr. who was a member of LDP confessed to
Rep. Jose Cojuangco (LDPs leader) that he voted for Bondoc even though Bondoc was a member of the NP. He
confessed that he believed in his conscience that Bondoc truly won the election. This resulted to
Camasuras expulsion from the LDP. Pineda then moved that they withdraw Camasura from the HRET. They further
prayed that a new election be held and that the new LDP representative be appointed in the HRET. This new
representative will be voting for Pineda in the reopening of the election contest. Camasura was then removed by
HRETs chairwoman Justice Ameurfina Herrera. Naturally, Bondoc questioned such action before the Supreme Court
(SC).

Pineda contends that the issue is already outside the jurisdiction of the Supreme Court because Camasuras removal
is an official act of Congress and by virtue of the doctrine of separation of powers, the judiciary may not interfere.
TAMPUS, MARY GRACE G. CONSTITUTIONAL LAW 1

ISSUE: Whether or not the Supreme Court may inquire upon the validity of the said act of the HRET without violating
the doctrine of separation of powers.

HELD:

Yes. The SC can settle the controversy in the case at bar without encroaching upon the function of the legislature
particularly a part thereof, HRET. The issue here is a judicial question. It must be noted that what is being
complained of is the act of HRET not the act of Congress. In here, when Camasura was rescinded by the tribunal, a
decision has already been made, members of the tribunal have already voted regarding the electoral contest
involving Pineda and Bondoc wherein Bondoc won. The LDP cannot withdraw their representative from the HRET
after the tribunal has already reached a decision. They cannot hold the same election since the issue has already
become moot and academic. LDP is merely changing their representative to change the outcome of the election.
Camasura should be reinstated because his removal was not due to a lawful or valid cause. Disloyalty to party is not
a valid cause for termination of membership in the HRET. Expulsion of Camasura violates his right to security of
tenure.

Infotech Foundation vs. COMELEC (G.R.No.159139, Jan.13, 2004)

FACTS

December 22, 1997, Congress enacted RA 8436 authorizing Comelec to use an automated election system for the
process of voting, counting votes, and canvassing/consolidating results of the national and local elections. It also
mandated the poll body to acquire automated counting machines, computer equipment, devices and materials and to
adopt new electoral forms and printing materials.

Initially intending to implement automation during May 11, 1998 presidential elections, COMELEC eventually decided
against full national implementation and limited the automation to ARMM. However, due to failure of machines to
read correctly some automated ballots in one town, poll body later ordered their manual count for the entire province
of Sulu. In the May 2001 elections, counting and canvassing of votes for both national and local positions were also
done manually, as no additional ACMs had been acquired to that electoral exercise allegedly because of time
constraints.
TAMPUS, MARY GRACE G. CONSTITUTIONAL LAW 1

Then, on January 24, 2003 PGMA issued EO 172 allocating the sum of P2.5 billion to fund the AES for the May 2004
elections. Upon request of COMELEC, PGMA authorized the release of an additional P500 million which then made
the Commission to issue an invitation to apply for eligibility to bid to interested offerors, vendors, suppliers or lessors.

Out of the 57 bidders, BAC found MPC and the Total Information Management Corporation (TIMC) eligible. For
technical evaluation, they were referred to the BACs Technical Working Group and DOST. In the report on
evaluation of technical proposals, DOST said that both MPC and TIMC had obtained number of failed marks in the
technical evaluation. Notwithstanding however these failures, Comelec en banc promulgated Resolution No. 6074
awarding the project to MPC thus prompting petitioner to write a letter to Comelec Chairman Benjamin Abalos, Sr.
protesting the award due to glaring irregularities in the manner in which the bidding process has been conducted
seeking re-bidding. However, Comelec Chairman rejected the protest, hence present petition.

ISSUE:

WON COMELEC gravely abused its discretion when, in the exercise of its administrative functions, it awarded to
MPC the contract for the 2nd phase of comprehensive AES

HELD:

Respondent Comelec came out with its en banc Resolution awarding the project to Respondent MPC even
before the BAC managed to issue its written report and recommendation. Thus, how could
petitioners have appealed the BACs recommendation or report to the head of the procuring entity (the
chairman of Comelec), when the Comelec en banc had already approved the award of the contract to MPC
even before petitioners learned of the BAC recommendation?

At that point the Commission en banc had already given its approval to the BAC Report alongwith the award to MPC.

To put it bluntly, the Comelec en banc itself made it legally impossible for petitioners to avail themselves of the
administrative remedy that the Commission is so impiously harping on.

There is no doubt that they had not been accorded the opportunity to availthemselves of the process provided under
Section 55 of RA 9184, according to which a protest against a decision of the BAC may be filed with the head of the
procuring entity. Comelec has not merely gravely abused its discretion in awarding the Contract for the automation of
the counting and canvassing of the ballots. It has also put at grave risk the holding of credible and peaceful elections
by shoddily accepting electronic hardware and software that admittedly failed to pass legally mandated technical
requirements. Inadequate as they are, the remedies it proffers post facto do not cure the grave abuse of discretion it
already committed (1) on April 15,2003, when it illegally made the award; and (2) "sometime" in May 2003 when it
executed the Contract for the purchase of defective machines and non-existent software -
from a noneligiblebidder.For these reasons, the Court finds it totally unacceptable and unconscionable to place itsimp
rimatur on this void and illegal transaction that seriously endangers the breakdown of our electoral system. For this
Court to cop-out and to close its eyes to these illegal transactions, while convenient, would be to abandon its
constitutional duty of safeguarding public interest. As a necessary consequence of such nullity and illegality, the
purchase of the machines and all appurtenances thereto including the still-to-be
produced (or in Comelecs words, to be"reprogrammed") software, as well as all the payments made therefor, have
TAMPUS, MARY GRACE G. CONSTITUTIONAL LAW 1

no basis whatsoever in law. The public funds expended pursuant to the void Resolution and Contract must therefore
be recovered from the payees and/or from the persons who made possible the illegal disbursements, without
prejudice to possible criminal prosecutions against them.

Tecson v. COMELEC

Fernando Poe, Jr. filed his certificate of candidacy for the position of President of Republic of Philippines in the
national elections. In his certificate of candidacy, FPJ, representing himself to be a natural-born citizen of the
Philippines.

A year after however, Victorino Fornier initiated a petition before the COMELEC to disqualify FPJ and for his
certificate of candidacy to be cancelled upon the thesis that FPJ made a material misrepresentation in his certificate
of candidacy by claiming to be a natural-born Filipino citizen when in truth, according to Fornier, his parents were
foreigners; his mother Bessie Kelley Poe, was an American, and his father Allan Poe was a Spanish national, being
the son of Lorenzo Pou, a Spanish subject. Granting, petitioner asseverated, that Allan Poe was a Filipino citizen, he
could not have transmitted his Filipino citizenship to FPJ, since FPJ is allegedly and illegitimate child of an alien
mother.
TAMPUS, MARY GRACE G. CONSTITUTIONAL LAW 1

After the presentation of documents, COMELEC dismissed the petition for lack of merit. Three days later, Fornier
filed a motion for reconsideration but was denied. Hence, this petition questioning the decision of COMELEC as well
as its jurisdiction.

ISSUE:

1. WON Supreme Court has jurisdiction over this case


2. WON FPJs Certificate of Candidacy be cancelled on the allegation that he is not a Filiipino citizen

HELD:

1. Yes.
Section 1, Article 8 of the 1987 Constitution provides that judicial power is vested in one Supreme Court and
in such lower courts as may be established by law which power includes the duty of the courts of justice to
settle actual controversies involving rights which are legally demandable and enforceable, and to determine
whether or not there has been grave abuse of discretion amounting to lack or excess of jurisdiction on the
part of any branch or instrumentality of the Government.

It is sufficiently clear therefore that the petition was aptly elevated to, and could well be taken cognizance of,
by this Court. A contrary view could be gross denial to our people of their fundamental right to be fully
informed, and to make a proper choice, on who could or should be elected to occupy the highest
government post in the land.
2. No, it should not be cancelled as FPJ is a natural-born Filipino citizen.
The term natural-born citizens, is defined to include those who are citizens of the Philippines from birth
without having to perform any act to acquire or perfect their Philippine citizenship.

The date, month and year of birth of FPJ appeared to be August 20, 1939 during the regime of 1935
Constitution. Through its history, four modes of acquiring citizenship-naturalization, jus soli, res judicata, and
jus sanguinishad been vogue. Only two: jus soli and jus sanguinis, could qualify a person to being a
natural born citizen of the Philippines.

The fact of the matter perhaps the most significant consideration is that the 1935 Constitution, the
fundamental law prevailing on the day, month and year of birth of respondent FPJ, can never be more
explicit than it is. Providing neither conditions nor distinctions,the Constitution states that among the citizens
of the Philippines are "those whose fathers are citizens of the Philippines." There utterly is no cogent
justification to prescribe conditions or distinctions where there are clearly none provided.

Any conclusion on the Filipino citizenship of Lorenzo Pou could only be drawn from the presumption that
having died in 1954 at 84 years old, Lorenzo would have been born sometime in the year 1870, when the
Philippines was under Spanish rule, and that San Carlos, Pangasinan, his place of residence upon his death
in 1954, in the absence of any other evidence, could have well been his place of residence before death,
such that Lorenzo Pou would have benefited from the "en masse Filipinization" that the Philippine bill had
effected in 1902. That citizenship (of Lorenzo Pou), if acquired, would thereby extend to his son, Allan F.
Poe, father of respondent FPJ. The 1935 Constitution, during which regime respondent FPJ has seen first
TAMPUS, MARY GRACE G. CONSTITUTIONAL LAW 1

light, confers citizenship to all persons whose fathers are Filipino citizens regardless of whether such
children are legitimate or illegitimate.

Mo Ya Lim Yao v. Commission on Immigration

FACTS

On 8 February 1961, Lau Yuen Yeung applied for a passport visa to enter the Philippines as a non-immigrant, for a
temporary visitor's visa to enter the Philippines. She was permitted to come into the Philippines on 13 March 1961.
On the date of her arrival, Asher Y, Cheng filed a bond in the amount of P1,000.00 to undertake, among others, that
said Lau Yuen Yeung would actually depart from the Philippines on or before the expiration of her authorized period
of stay in this country or within the period as in his discretion the Commissioner of Immigration. After repeated
extensions, she was allowed to stay in the Philippines up to 13 February 1962. On 25 January 1962, she contracted
marriage with Moy Ya Lim Yao alias Edilberto Aguinaldo Lim an alleged Filipino citizen. Because of the contemplated
TAMPUS, MARY GRACE G. CONSTITUTIONAL LAW 1

action of the Commissioner of Immigration to confiscate her bond and order her arrest and immediate deportation,
after the expiration of her authorized stay, she brought an action for injunction with preliminary injunction. The Court
of First Instance of Manila (Civil Case 49705) denied the prayer for preliminary injunction. Moya Lim Yao and Lau
Yuen Yeung appealed.

ISSUE:

Whether Lau Yuen Yeung ipso facto became a Filipino citizen upon her marriage to a Filipino citizen.

HELD:
Under Section 15 of Commonwealth Act 473, an alien woman marrying a Filipino, native born or naturalized,
becomes ipso facto a Filipina provided she is not disqualified to be a citizen of the Philippines under Section 4
of the same law. Likewise, an alien woman married to an alien who is subsequently naturalized here follows the
Philippine citizenship of her husband the moment he takes his oath as Filipino citizen, provided that she does not
suffer from any of the disqualifications under said Section 4. Whether the alien woman requires to undergo the
naturalization proceedings, Section 15 is a parallel provision to Section 16. Thus, if the widow of an applicant for
naturalization as Filipino, who dies during the proceedings, is not required to go through a naturalization proceedings,
in order to be considered as a Filipino citizen hereof, it should follow that the wife of a living Filipino cannot be denied
the same privilege. Everytime the citizenship of a person is material or indispensible in a judicial or administrative
case, Whatever the corresponding court or administrative authority decides therein as to such citizenship is generally
not considered as res judicata, hence it has to be threshed out again and again as the occasion may demand. Lau
Yuen Yeung, was declared to have become a Filipino citizen from and by virtue of her marriage to Moy Ya
Lim Yao alias Edilberto Aguinaldo Lim, a Filipino citizen of 25 January 1962.

Valles v. COMELEC

FACTS
Rosalind Ybasco Lopez was born on May 16, 1934 in Australia to a Filipino father and an Australian mother. In 1949,
at the age of fifteen, she left Australia and came to settle in the Philippines, where she later married a Filipino and
has since then participated in the electoral process not only as a voter but as a candidate, as well. In the May 1998
elections, she ran for governor but Valles filed a petition for her disqualification as candidate on the ground that she is
TAMPUS, MARY GRACE G. CONSTITUTIONAL LAW 1

an Australian.

ISSUE:

Whether or not Rosalind is an Australian or a Filipino

HELD:

The Philippine law on citizenship adheres to the principle of jus sanguinis. Thereunder, a child follows the nationality
or citizenship of the parents regardless of the place of his/her birth, as opposed to the doctrine of jus soli which
determines nationality or citizenship on the basis of place of birth.

Rosalind Ybasco Lopez was born a year before the 1935 Constitution took into effect and at that time, what served
as the Constitution of the Philippines were the principal organic acts by which the United States governed the country.
These were the Philippine Bill of July 1, 1902 and the Philippine Autonomy Act of Aug. 29, 1916, also known as the
Jones Law.

Under both organic acts, all inhabitants of the Philippines who were Spanish subjects on April 11, 1899 and resided
therein including their children are deemed to be Philippine citizens. Private respondents father, Telesforo Ybasco,
was born on Jan. 5, 1879 in Daet, Camarines Norte.... Thus, under the Philippine Bill of 1902 and the Jones Law,
Telesforo Ybasco was deemed to be a Philippine citizen. By virtue of the same laws, which were the laws in force at
the time of her birth, Telesforos daughter, herein private respondent Rosalind Ybasco Lopez, is likewise a citizen of
the Philippines.

The signing into law of the 1935 Philippine Constitution has established the principle of jus sanguinis as basis for the
acquisition of Philippine citizenship, xxx

So also, the principle of jus sanguinis, which confers citizenship by virtue of blood relationship, was subsequently
retained under the 1973 and 1987 Constitutions. Thus, the herein private respondent, Rosalind Ybasco Lopez, is a
Filipino citizen, having been born to a Filipino father. The fact of her being born in Australia is not tantamount to her
losing her Philippine citizenship. If Australia follows the principle of jus soli, then at most, private respondent can also
claim Australian citizenship resulting to her possession of dual citizenship.
TAMPUS, MARY GRACE G. CONSTITUTIONAL LAW 1

Bengson III v. HRET

FACTS
Cruz was a natural-born citizen of the Philippines. He was born in San Clemente, Tarlac, on April 27, 1960, of Filipino
parents. The fundamental law then applicable was the 1935 Constitution. On November 5, 1985, however,
respondent Cruz enlisted in the United States Marine Corps and, without the consent of the Republic of the
Philippines, took an oath of allegiance to the United States. As a consequence, he lost his Filipino citizenship for
under Commonwealth Act No. 63, Section 1(4), a Filipino citizen may lose his citizenship by, among others,
"rendering service to or accepting commission in the armed forces of a foreign country."

Whatever doubt that remained regarding his loss of Philippine citizenship was erased by his naturalization as a U.S.
citizen on June 5, 1990, in connection with his service in the U.S. Marine Corps.
On March 17, 1994, respondent Cruz reacquired his Philippine citizenship through repatriation under Republic Act No.
2630. 3 He ran for and was elected as the Representative of the Second District of Pangasinan in the May 11, 1998
elections. He won by a convincing margin of 26,671 votes over petitioner Antonio Bengson III, who was then running
for reelection. Subsequently, petitioner filed a case for Quo Warranto Ad Cautelam with respondent House of
Representatives Electoral Tribunal (HRET) claiming that respondent Cruz was not qualified to become a member of
the House of Representatives since he is not a natural-born citizen as required under Article VI, Section 6 of the
Constitution.

On March 2, 2000, the HRET rendered its decision 5 dismissing the petition, for quo warranto and declaring
respondent Cruz the duly elected Representative of the Second District of Pangasinan in the May 1998 elections.
The HRET likewise denied petitioner's motion for reconsideration of the decision.

ISSUE:
WON Cruz, a natural born Filipino, who became an American citizen, can still be considered a natural-born Filipino
upon his re-acquisition of PH citizenship.

HELD:
Yes, he can still be considered as natural-born.

The 1987 Constitution enumerates who are Filipino citizens as follows:


(1) Those who are citizens of the Philippines at the time of the adoption of this Constitution;
(2) Those whose fathers or mothers are citizens of the Philippines;
(3) Those born before January 17, 1973 of Filipino mothers, who elect Philippine citizenship upon reaching the age of
majority, and
(4) Those who are naturalized in accordance with law.

There are two ways of acquiring citizenship: (1) by birth, and (2) by naturalization. These ways of acquiring
citizenship correspond to the two kinds of citizens: the natural-born citizen, and the naturalized citizen. A person who
TAMPUS, MARY GRACE G. CONSTITUTIONAL LAW 1

at the time of his birth is a citizen of a particular country, is a natural-born citizen thereof. As defined in the same
Constitution, natural-born citizens "are those citizens of the Philippines from birth without having to perform any act to
acquire or perfect his Philippine citizenship."

On the other hand, naturalized citizens are those who have become Filipino citizens through naturalization, generally
under Commonwealth Act No. 473, otherwise known as the Revised Naturalization Law, which repealed the former
Naturalization Law (Act No. 2927), and by Republic Act No. 530. To be naturalized, an applicant has to prove that he
possesses all the qualifications and none of the disqualifications provided by law to become a Filipino citizen. The
decision granting Philippine citizenship becomes executory only after two (2) years from its promulgation when the
court is satisfied that during the intervening period, the applicant has (1) not left the Philippines; (2) has dedicated
himself to a lawful calling or profession; (3) has not been convicted of any offense or violation of Government
promulgated rules; or (4) committed any act prejudicial to the interest of the nation or contrary to any Government
announced policies.

Filipino citizens who have lost their citizenship may however reacquire the same in the manner provided by law.
Commonwealth Act No. 63 (CA No. 63), enumerates the three modes by which Philippine citizenship may be
reacquired by a former citizen: (1) by naturalization, (2) by repatriation, and (3) by direct act of Congress.

Naturalization is a mode for both acquisition and reacquisition of Philippine citizenship. As a mode of initially
acquiring Philippine citizenship, naturalization is governed by Commonwealth Act No. 473, as amended. On the other
hand, naturalization as a mode for reacquiring Philippine citizenship is governed by Commonwealth Act No. 63.
Under this law, a former Filipino citizen who wishes to reacquire Philippine citizenship must possess certain
qualifications and none of the disqualifications mentioned in Section 4 of C.A. 473.
Repatriation, on the other hand, may be had under various statutes by those who lost their citizenship due to: (1)
desertion of the armed forces; (2) service in the armed forces of the allied forces in World War II; (3) service in the
Armed Forces of the United States at any other time; (4) marriage of a Filipino woman to an alien; and (5) political
and economic necessity.

As distinguished from the lengthy process of naturalization, repatriation simply consists of the taking of an oath of
allegiance to the Republic of the Philippines and registering said oath in the Local Civil Registry of the place where
the person concerned resides or last resided. Moreover, repatriation results in the recovery of the original nationality.
This means that a naturalized Filipino who lost his citizenship will be restored to his prior status as a naturalized
Filipino citizen. On the other hand, if he was originally a natural-born citizen before he lost his Philippine citizenship,
he will be restored to his former status as a natural-born Filipino.

In respondent Cruz's case, he lost his Filipino citizenship when he rendered service in the Armed Forces of the
United States. However, he subsequently reacquired Philippine citizenship under R.A. No. 2630. Having thus taken
the required oath of allegiance to the Republic and having registered the same in the Civil Registry of Magantarem,
Pangasinan in accordance with the aforecited provision, respondent Cruz is deemed to have recovered his original
status as a natural-born citizen, a status which he acquired at birth as the son of a Filipino father. It bears stressing
that the act of repatriation allows him to recover, or return to, his original status before he lost his Philippine
citizenship.
TAMPUS, MARY GRACE G. CONSTITUTIONAL LAW 1

Co v. HRET

FACTS

On May 11, 1987, the congressional election for the second district of Northern Samar was held.
Among the candidates who vied for the position of representative in the second legislative district of Northern Samar
are the petitioners, Sixto Balinquit and Antonio Co and the private respondent, Jose Ong, Jr. Respondent Ong was
proclaimed the duly elected representative of the second district of Northern Samar.
The petitioners filed election protests against the private respondent premised on the following grounds:
1) Jose Ong, Jr. is not a natural born citizen of the Philippines; and
2) Jose Ong, Jr. is not a resident of the second district of Northern Samar.
The HRET, in its decision dated November 6, 1989, found for the private respondent.
A motion for reconsideration was filed by the petitioners on November 12, 1989. This was, however, denied by the
HRET.

Hence, these petitions for certiorari.

ISSUE:

WON Jose Ong is a natural-born citizen of the Philippines?


TAMPUS, MARY GRACE G. CONSTITUTIONAL LAW 1

HELD:

Yes, Jose Ong is a natural-born citizen.


The records show that in the year 1895, the private respondent's grandfather, Ong Te, arrived in the Philippines from
China. Ong Te established his residence in the municipality of Laoang, Samar on land which he bought from the
fruits of hard work. As a resident of Laoang, Ong Te was able to obtain a certificate of residence from the then
Spanish colonial administration.

The father of the private respondent, Jose Ong Chuan was born in China in 1905. He was brought by Ong Te to
Samar in the year 1915. Jose Ong Chuan spent his childhood in the province of Samar. In Laoang, he was able to
establish an enduring relationship with his neighbors, resulting in his easy assimilation into the community.

As Jose Ong Chuan grew older in the rural and seaside community of Laoang, he absorbed Filipino cultural values
and practices. He was baptized into Christianity. As the years passed, Jose Ong Chuan met a natural born-Filipina,
Agripina Lao. The two fell in love and, thereafter, got married. The couple bore eight children, one of whom is the
private respondent who was born in 1948.The private respondent's father never emigrated from this country. He
decided to put up a hardware store and shared and survived the vicissitudes of life in Samar. The business
prospered. Expansion became inevitable.

In the meantime, the father of the private respondent, unsure of his legal status and in an unequivocal affirmation of
where he cast his life and family, filed with the Court of First Instance of Samar of application for naturalization. CFI of
Samar, after trial, declared Jose Ong Chuan as Filipino citizen which, thereafter, Jose Ong Chuan took his oath of
allegiance and was issued his certificate of naturalization.

At the time Jose Ong Chuan took his oath, the private respondent then a minor of nine years was finishing his
elementary education in the province of Samar. There is nothing in the records to differentiate him from other
Filipinos insofar as the customs and practices of the local populace were concerned.

After completing his elementary education, the private respondent, in search for better education, went to Manila in
order to acquire his secondary and college education. In the meantime, another misfortune was suffered by the
family in 1975 when a fire gutted their second house in Laoang, Samar. The respondent's family constructed still
another house, this time a 16-door apartment building, two doors of which were reserved for the family. The private
respondent graduated from college, and thereafter took and passed the CPA Board Examinations.

Since employment opportunities were better in Manila, the respondent looked for work here. He found a job in the
Central Bank of the Philippines as an examiner. Later, however, he worked in the hardware business of his family in
Manila. In 1971, his elder brother, Emil, was elected as a delegate to the 1971 Constitutional Convention. His status
as a natural born citizen was challenged. Parenthetically, the Convention which in drafting the Constitution removed
the unequal treatment given to derived citizenship on the basis of the mother's citizenship formally and solemnly
declared Emil Ong, respondent's full brother, as a natural born Filipino.
TAMPUS, MARY GRACE G. CONSTITUTIONAL LAW 1

For the elections of 1984 and 1986, Jose Ong, Jr. registered himself as a voter of Laoang, Samar, and
correspondingly, voted there during those elections. The private respondent after being engaged for several years in
the management of their family business decided to be of greater service to his province and ran for public office.
Hence, when the opportunity came in 1987, he ran in the elections for representative in the second district of
Northern Samar.
Mr. Ong was overwhelmingly voted by the people of Northern Samar as their representative in Congress.

There is no dispute that the respondent's mother was a natural born Filipina at the time of her marriage. Crucial to
this case is the issue of whether or not the respondent elected or chose to be a Filipino citizen.
Election becomes material because Section 2 of Article IV of the Constitution accords natural born status to children
born of Filipino mothers before January 17, 1973, if they elect citizenship upon reaching the age of majority.

To expect the respondent to have formally or in writing elected citizenship when he came of age is to ask for the
unnatural and unnecessary. The reason is obvious. He was already a citizen. Not only was his mother a natural born
citizen but his father had been naturalized when the respondent was only nine (9) years old. He could not have
divined when he came of age that in 1973 and 1987 the Constitution would be amended to require him to have filed a
sworn statement in 1969 electing citizenship inspite of his already having been a citizen since 1957. In 1969, election
through a sworn statement would have been an unusual and unnecessary procedure for one who had been a citizen
since he was nine years old.

The filing of sworn statement or formal declaration is a requirement for those who still have to elect citizenship. For
those already Filipinos when the time to elect came up, there are acts of deliberate choice which cannot be less
binding. Entering a profession open only to Filipinos, serving in public office where citizenship is a qualification, voting
during election time, running for public office, and other categorical acts of similar nature are themselves formal
manifestations of choice for these persons.

An election of Philippine citizenship presupposes that the person electing is an alien. Or his status is doubtful
because he is a national of two countries. There is no doubt in this case about Mr. Ong's being a Filipino when he
turned twenty-one (21). We repeat that any election of Philippine citizenship on the part of the private respondent
would not only have been superfluous but it would also have resulted in an absurdity. How can a Filipino citizen elect
Philippine citizenship?

Cabiling Ma v. Commissioner (G.R. No.183133, July 26, 2010)

FACTS
TAMPUS, MARY GRACE G. CONSTITUTIONAL LAW 1

Balgamelo Cabiling Ma, Felix Cabiling Ma, Valeriano Cabiling Ma, Lechi Ann Ma, Arceli Ma, Nicolas Ma, and Isidro
Ma are the children of Felix Ma, a Taiwanese and Dolores Cabiling, a Filipina.

Records reveal that the petitioners were all born under aegis of the 1935 Philippine Constitution and were, together
with their other siblings, raised in the Philippines and have resided in this country for almost 60 years; spent their
whole lives, studied and received their primary and secondary education in the country; they do not speak nor
understand Chinese nor have they set foot in Taiwan or even traveled abroad, and they have already raised their
respective families in the Philippines.

During their age of minority, they secured ACRs (Alien Certificate of Registration) and immediately upon reaching the
age of majority (21), claimed PH citizenship in accordance with Section 1(4), Article 4 of the 1935 Constitution which
provides that those whose mothers are citizens of the Philippines and upon reaching the age of majority, elect PH
citizenship are citizens of the Philippines. Thus on August 15, 1969, Felix, Jr. executed his affidavit of election of PH
citizenship and took his oath of allegiance.

Having taken their oath of allegiance as PH citizens, petitioners, however, failed to have the necessary documents
registered in the civil registry as required under Section 1 of CA 625 and it was only after more than 30 years after
they elected PH citizenship that they did so. Then on February 16, 2004, Bureau of Immigration received a
complaint-affidavit of a certain Mat Catral alleging that Felix Ma and his 7 children are undesirable and overstaying
aliens.

After Felix Ma and his 7 children were afforded the opportunity to refute the allegations, Board of Commissioners of
Bureau of Immigration rendered then a judgment finding that Felix Ma and his children violated CA 613.

ISSUE:
WON the petitioners are Filipino citizens

HELD:

No direct answer but simply given the chance to register themselves considering they have Filipino mother.

The 1935 Constitution declares as citizens of the Philippines those whose mothers are citizens of the Philippines and
elect Philippine citizenship upon reaching the age of majority. The mandate states:
Section 1. The following are citizens of the Philippines:
(1) ...;

xxx xxx xxx


(4) Those whose mothers are citizens of the Philippines and, upon reaching the age of majority, elect Philippine
citizenship.

CA 625 as well provides the manner of electing Philippine citizenship, to wit:


TAMPUS, MARY GRACE G. CONSTITUTIONAL LAW 1

Section 1. The option to elect Philippine citizenship in accordance with subsection (4), Section 1, Article IV, of the
Constitution shall be expressed in a statement to be signed and sworn to by the party concerned before any officer
authorized to administer oaths, and shall be filed with the nearest civil registry. The said party shall accompany the
aforesaid statement with the oath of allegiance to the Constitution and the Government of the Philippines.

The statutory formalities of electing Philippine citizenship are: (1) a statement of election under oath; (2) an oath of
allegiance to the Constitution and Government of the Philippines; and (3) registration of the statement of election and
of the oath with the nearest civil registry.
In the present case, the petitioners complied with the 1st and 2nd requirements upon reaching the age of majority. It
was only the registration of documents of election with civil registry that was belatedly done. It is then to be ruled that
under the facts, the right to elect PH citizenship has not been lost and they should be allowed to complete the
statutory requirements for such election.

We are not prepared to state that the mere exercise of suffrage, being elected public official, continuous and
uninterrupted stay in the Philippines, and other similar acts showing exercise of Philippine citizenship can take the
place of election of citizenship. What we now say is that where, as in petitioners case, the election of citizenship has
in fact been done and documented within the constitutional and statutory timeframe, the registration of the
documents of election beyond the frame should be allowed if in the meanwhile positive acts of citizenship have
publicly, consistently, and continuously been done. The actual exercise of Philippine citizenship, for over half a
century by the herein petitioners, is actual notice to the Philippine public which is equivalent to formal registration of
the election of Philippine citizenship.

We are guided by this evolvement from election of Philippine citizenship upon reaching the age of majority under the
1935 Philippine Constitution to dispensing with the election requirement under the 1973 Philippine Constitution to
express classification of these children as natural-born citizens under the 1987 Constitution towards the conclusion
that the omission of the 1941 statutory requirement of registration of the documents of election should not result in
the obliteration of the right to Philippine citizenship.

Having a Filipino mother is permanent. It is the basis of the right of the petitioners to elect Philippine citizenship.
Petitioners elected Philippine citizenship in form and substance. The failure to register the election in the civil registry
should not defeat the election and resultingly negate the permanent fact that they have a Filipino mother. The lacking
requirements may still be complied with subject to the imposition of appropriate administrative penalties, if any. The
documents they submitted supporting their allegations that they have already registered with the civil registry,
although belatedly, should be examined for validation purposes by the appropriate agency, in this case, the Bureau
of Immigration. Other requirements embodied in the administrative orders and other issuances of the Bureau of
Immigration and the Department of Justice shall be complied with within a reasonable time.
TAMPUS, MARY GRACE G. CONSTITUTIONAL LAW 1

Republic v. de la Rosa (G.R. No. 104654, June, 1994)

FACTS
On Sept. 20, 1991, Frivaldo filed a petition for naturalization, which said petition was granted then set to hearing and
then directed for publication of the order and petition in Official Gazette and a newspaper of general circulation, for 3
consecutive weeks, the last publication of which should be at least 6 months before the said hearing. However,
Frivaldo filed a Motion to set hearing ahead of schedule where he manifested his intention to run for public office in
May 1992 elections. He alleged that the deadline for filing the certificate of candidacy was March 15, one day before
the scheduled hearing. Motion was granted and the hearing was moved. It is also to be noted that the order was not
published nor a copy posted.

Hearing proceeded with Frivaldo as the sole witness. He then submitted various documentary evidence:
-affidavit of publication of order
-certification of publication of order
-notice of hearing of petitions
-certification of appreciation issued by Rotary Club of Davao
-Photocopy of plaque of appreciation issued by Republican College
-Certification of Naturalization

Six days later, Judge rendered a decision granting the petition of Frivaldo and on that same day, Frivaldo was
allowed to take his oath of allegiance before the Judge.
However, a certain Quiterio Hermo alleged that the proceedings were tainted with jurisdictional defects, and prayed
for new trial to conform with the requirements of the Naturalization Law. Republic of the Philippines as well
interposed an appeal through the Solicitor General, hence present petition.

ISSUE:
TAMPUS, MARY GRACE G. CONSTITUTIONAL LAW 1

WON Frivaldo was duly re-admitted to his citizenship as Filipino

HELD:
No, he was not. The supreme court ruled that Private respondent is declared NOT a citizen of the Philippines and
therefore disqualified from continuing to serve as governor of the Province of Sorsogon.
Frivaldo, having opted to reacquire PH citizenship through naturalization under Revised Naturalization Law is duty
bound to follow the procedure prescribed by the said law. It is not for an applicant to decide for himself and to select
the requirements which he believes, even sincerely, are applicable to his case and discard those which he believes
are inconvenient or merely of nuisance value. The law does not distinguish between an applicant who was formerly a
Filipino citizen and one who was never such a citizen. It does not provide a special procedure for the reacquisition of
Philippine citizenship by former Filipino citizens akin to the repatriation of a woman who had lost her Philippine
citizenship by reason of her marriage to an alien.

The trial court never acquired jurisdiction to hear the petition for naturalization of private respondent. The
proceedings conducted, the decision rendered and the oath of allegiance taken therein, are null and void for failure to
comply with the publication and posting requirements under the Revised Naturalization Law.

The petition for naturalization lacks several allegations required by Sections 2 and 6 of the Revised Naturalization
Law, particularly: (1) that the petitioner is of good moral character; (2) that he resided continuously in the Philippines
for at least ten years; (3) that he is able to speak and write English and any one of the principal dialects; (4) that he
will reside continuously in the Philippines from the date of the filing of the petition until his admission to Philippine
citizenship; and (5) that he has filed a declaration of intention or if he is excused from said filing, the justification
therefor. The absence of such allegations is fatal to the petition.

Frivaldo v. COMELEC (257 SCRA 731)

FACTS:

Frivaldo filed his Certificate of Candidacy for the office of Governor of Sorsogon however, 4 days after filing his COC,
another candidate, Raul Lee, filed a petition against Frivaldo praying for him to be disqualified from seeking or
holding any public office or position by reason of not yet being a citizen of the Philippines and that his COC be
cancelled which was granted. Frivaldo filed for a motion for reconsideration but remained unacted up until May 8,
1995 elections so his candidacy continued and he was voted for during the elections held on that date.

Several days after, canvassing of votes were completed and a certificate of votes was issued showing the number of
votes each candidate obtained. Frivaldo garnered the highest number of votes but since there was a disqualification
petition against him hence the Provincial Board of Canvassers proclaimed Raul Lee as the winning gubernatorial
candidate garnering the 2nd number of highest votes. Frivaldo, therefore, filed a new petition praying for the
annulment of the proclamation of Lee as the winning candidate contending that it should be him who should be
declared the winning candidate in the gubernatorial election considering having garnered the highest number of votes
TAMPUS, MARY GRACE G. CONSTITUTIONAL LAW 1

and having reacquired his Filipino citizenship by repatriation under the provision of PD 725 qualifying him to hold
office of the governor of Sorsogon.

First Division of Comelec granted the petition. Lee filed a motion for reconsideration but was denied by the Comelec
en banc, hence the present petition.

ISSUE:
WON the repatriation of Frivaldo is valid and legal? If so, did it seasonably cure his lack of citizenship as to qualify
him to be proclaimed and to hold the Office of the Governor?

HELD:

Yes, the repatriation of Frivaldo is valid and legal and cured his lack of citizenship qualifying him to be proclaimed
and hold Office of the Governor.

Philippine citizenship is an indispensable requirement for holding an elective public office, and the purpose of the
citizenship qualification is none other than to ensure that no alien, i.e., no person owing allegiance to another nation,
shall govern our people and our country or a unit of territory thereof. Now, an official begins to govern or to discharge
his functions only upon his proclamation and on the day the law mandates his term of office to begin. Since Frivaldo
re-assumed his citizenship on June 30, 1995 the very day the term of office of governor (and other elective
officials) began he was therefore already qualified to be proclaimed, to hold such office and to discharge
the functions and responsibilities thereof as of said date. In short, at that time, he was already qualified to
govern his native Sorsogon. This is the liberal interpretation that should give spirit, life and meaning to our law on
qualifications consistent with the purpose for which such law was enacted. So too, even from a literal (as
distinguished from liberal) construction, it should be noted that Section of the Local Government Code speaks of
"Qualifications" of "ELECTIVE OFFICIALS", not of candidates.

But perhaps the more difficult objection was the one raised during the oral argument to the effect that the citizenship
qualification should be possessed at the time the candidate (or for that matter the elected official) registered as a
voter. After all, Section 39, apart from requiring the official to be a citizen, also specifies as another item of
qualification, that he be a "registered voter". And, under the law 35 a "voter" must be a citizen of the Philippines.

The answer to this problem again lies in discerning the purpose of the requirement. If the law intended the citizenship
qualification to be possessed prior to election consistent with the requirement of being a registered voter, then it
would not have made citizenship a SEPARATE qualification. It therefore stands to reason that the law intended
CITIZENSHIP to be a qualification distinct from being a VOTER, even if being a voter presumes being a citizen first.
It also stands to reason that the voter requirement was included as another qualification (aside from "citizenship"),
not to reiterate the need for nationality but to require that the official be registered as a voter IN THE AREA OR
TERRITORY he seeks to govern, i.e., the law states: "a registered voter in the barangay, municipality, city, or
province . . . where he intends to be elected." It should be emphasized that the Local Government Code requires an
elective official to be a registered voter. It does not require him to vote actually. Hence, registration not the actual
TAMPUS, MARY GRACE G. CONSTITUTIONAL LAW 1

voting is the core of this "qualification". In other words, the law's purpose in this second requirement is to ensure
that the prospective official is actually registered in the area he seeks to govern and not anywhere else. Ever
since, Frivaldo has been adamant that was and has always been a registered voter of Sorsogon which was not
disputed, hence, it is clear that Frivaldo, is indeed, a registered voter in the province where he intended to be elected.

But to remove all doubts on this important issue, we also hold that the repatriation of Frivaldo RETROACTED to the
date of the filing of his application on August 17, 1994.

It is true that under the Civil Code of the Philippines, "(l)aws shall have no retroactive effect, unless the contrary is
provided." But there are settled exceptions to this general rule, such as when the statute is CURATIVE or
REMEDIAL in nature or when it CREATES NEW RIGHTS. By their very nature, curative statutes are retroactive . . .
(and) reach back to past events to correct errors or irregularities and to render valid and effective attempted acts
which would be otherwise ineffective for the purpose the parties intended."

A reading of P.D. 725 immediately shows that it creates a new right, and also provides for a new remedy, thereby
filling certain voids in our laws. Thus, in its preamble, P.D. 725 expressly recognizes the plight of "many Filipino
women (who) had lost their Philippine citizenship by marriage to aliens" and who could not, under the existing law
(C.A. No. 63, as amended) avail of repatriation until "after the death of their husbands or the termination of their
marital status" and who could neither be benefited by the 1973 Constitution's new provision allowing "a Filipino
woman who marries an alien to retain her Philippine citizenship . . ." because "such provision of the new Constitution
does not apply to Filipino women who had married aliens before said constitution took effect." On the other hand,
said statute also provided a new remedy and a new right in favor of other "natural born Filipinos who (had) lost their
Philippine citizenship but now desire to re-acquire Philippine citizenship", because prior to the promulgation of P.D.
725 such former Filipinos would have had to undergo the tedious and cumbersome process of naturalization, but with
the advent of P.D. 725 they could now re-acquire their Philippine citizenship under the simplified procedure of
repatriation.

That is, the repatriation granted to Frivaldo on June 30, 1995 can and should be made to take effect as of date of his
application. As earlier mentioned, there is nothing in the law that would bar this or would show a contrary intention on
the part of the legislative authority; and there is no showing that damage or prejudice to anyone, or anything unjust or
injurious would result from giving; retroactivity to his repatriation. Neither has Lee shown that there will result the
impairment of any contractual obligation, disturbance of any vested right or breach of some constitutional guaranty.
Being a former Filipino who has served the people repeatedly, Frivaldo deserves a liberal interpretation of Philippine
laws and whatever defects there were in his nationality should now be deemed mooted by his repatriation.
TAMPUS, MARY GRACE G. CONSTITUTIONAL LAW 1

Tabasa v. Court of Appeals (G.R.No. 125793, August 29, 2006)

FACTS

Joevanie Arellano Tabasa was a natural-born citizen of the Philippines. In 1968, when petitioner was seven years old,
his father, Rodolfo Tabasa, became a naturalized citizen 5 of the United States. By derivative naturalization
(citizenship derived from that of another as from a person who holds citizenship by virtue of naturalization ), petitioner
also acquired American citizenship.

Tabasa arrived in the Philippines on August 3, 1995, and was admitted as a "balikbayan" for one year. Thereafter,
petitioner was arrested and detained by agent Wilson Soluren of the BID on May 23, 1996, pursuant to BID Mission
Order. Petitioner was investigated by Special Prosecutor Atty. Edy D. Donato at the Law and Investigation Division of
the BID on May 28, 1996; and on the same day, Tabasa was accused of violating Section 8, Chapter 3, Title 1, Book
3 of the 1987 Administrative Code. BID ordered petitioner's deportation to his country of origin, the United States, on
May 29, 1996, in the following summary deportation order. Because of this, Petitioner filed before the CA a Petition
for Habeas Corpus with Preliminary Injunction and/or Temporary Restraining Order. Tabasa alleged that he was not
afforded due process; that no warrant of arrest for deportation may be issued by immigration authorities before a final
order of deportation is made; that no notice of the cancellation of his passport was made by the U.S. Embassy; that
he is entitled to admission or to a change of his immigration status as a non-quota immigrant because he is married
to a Filipino citizen as provided in Section 13, paragraph (a) of the Philippine Immigration Act of 1940; and that he
was a natural-born citizen of the Philippines prior to his derivative naturalization when he was seven years old due to
the naturalization of his father, Rodolfo Tabasa, in 1968. At the time Tabasa filed said petition, he was already 35
years old.

However, on June 13, 1996, petitioner filed a Supplemental Petition alleging that he had acquired Filipino citizenship
by repatriation in accordance with Republic Act No. 8171 (RA 8171), and that because he is now a Filipino citizen, he
cannot be deported or detained by the respondent Bureau.

CA denied Tabasas petition on the ground that he had not legally and successfully acquired by repatriation his
Filipino citizenship by derivative naturalization when his father was naturalized in 1968, there is no evidence to show
that he lost his PH citizenship on account of political or economic necessity. CA concluded that his only reason to
want to reacquire PH citizenship is to avoid criminal prosecution in US. The Court therefore ruled against Tabasa.
Hence, this petition.

ISSUE:
WON Tabasa has validly acquired Philippine Citizenship under RA 8171?

HELD:

No. He has not validly acquired PH citizenship under RA 8171.


TAMPUS, MARY GRACE G. CONSTITUTIONAL LAW 1

RA 8171, "An Act Providing for the Repatriation of Filipino Women Who Have Lost Their Philippine Citizenship by
Marriage to Aliens and of Natural-Born Filipinos," was enacted on October 23, 1995. It provides for the repatriation of
only two (2) classes of persons, viz:

Filipino women who have lost their Philippine citizenship by marriage to aliens and natural-born Filipinos
who have lost their Philippine citizenship, including their minor children, on account of political or economic
necessity, may reacquire Philippine citizenship through repatriation which Tabasa does not qualify.

Tabasa was no longer a minor at the time of his repatriation as the privilege under RA 8171 belongs to children who
are of minor age at the time of the filing of the petition for repatriation. Neither can he be a natural-born Filipino who
left the country due to political or economic necessity. Clearly, he lost his PH citizenship by operation of law and not
due to his political or economic reasons as it was his father who could have been motivated by economic or political
reasons in deciding to apply for naturalization. The decision was his parents and not his. The privilege of repatriation
under RA 8171 is extended directly to the natural born Filipinos who could prove that they acquired citizenship of a
foreign country due to political and economic reasons, and extended indirectly to the minor children at the time of
repatriation.

In sum, petitioner is not qualified to avail himself of repatriation under RA 8171. However, he can possibly reacquire
Philippine citizenship by availing of the Citizenship Retention and Re-acquisition Act of 2003 (Republic Act No. 9225)
by simply taking an oath of allegiance to the Republic of the Philippines.

Even if we concede that petitioner Tabasa can avail of the benefit of RA 8171, still he failed to follow the procedure
for reacquisition of Philippine citizenship. He has to file his petition for repatriation with the Special Committee on
Naturalization (SCN), which was designated to process petitions for repatriation pursuant to Administrative Order No.
285 (A.O. No. 285) dated August 22, 1996. What petitioner simply did was that he took his oath of allegiance to the
Republic of the Philippines; then, executed an affidavit of repatriation, which he registered, together with the
certificate of live birth, with the Office of the Local Civil Registrar of Manila. The said office subsequently issued him a
certificate of such registration. At that time, the SCN was already in place and operational by virtue of the June 8,
1995 Memorandum issued by President Fidel V. Ramos. Although A.O. No. 285 designating the SCN to process
petitions filed pursuant to RA 8171 was issued only on August 22, 1996, it is merely a confirmatory issuance.
TAMPUS, MARY GRACE G. CONSTITUTIONAL LAW 1

Mercado v. Manzano (307 SCRA 630)

FACTS

Petitioner Ernesto S. Mercado and private respondent Eduardo B. Manzano were candidates for vice mayor of the
City of Makati in the May 11, 1998 elections. The other one was Gabriel V. Daza III. The results of the election were
as follows:

Eduardo B. Manzano 103,853


Ernesto S. Mercado 100,894
Gabriel V. Daza III 54,275

The proclamation of private respondent was suspended in view of a pending petition for disqualification filed by a
certain Ernesto Mamaril who alleged that private respondent was not a citizen of the Philippines but of the United
States. In its resolution, dated May 7, 1998, the Second Division of the COMELEC granted the petition of Mamaril
and ordered the cancellation of the certificate of candidacy of private respondent on the ground that he is a dual
citizen and, under 40(d) of the Local Government Code, persons with dual citizenship are disqualified from running
TAMPUS, MARY GRACE G. CONSTITUTIONAL LAW 1

for any elective position. Manzano filed a motion for reconsidering which remained pending even until after the
holding of elections.

Accordingly thereafter, pursuant to Omnibus Resolution 3044, COMELEC tabulated the votes cast for vice mayor but
suspended the proclamation of winner which Mercado sought to intervene in the case for disqualification of Manzano
which was then opposed by Manzano himself.

COMELEC then released a resolution stating and proclaiming that Manzano is the winning candidate for vice-mayor
of Makati City and pursuant to the said resolution, Mercado filed for petition for certiorari and to declare Manzano as
disqualified to hold office as he was allegedly holding dual citizenship which disqualifies one from running for any
elective local position.

ISSUE:

WON Manzano is disqualified from the position of vice-mayor for which he filed in his COC. Is he eligible for the office
he seeks?

HELD:

No, he is not disqualified as what is prohibited is dual allegiance and not dual citizenship.

Dual citizenship is different from dual allegiance. Dual citizenship arises as a result of concurrent application for
different laws of two or more states where a person is simultaneously considered a national by the said states. Dual
allegiance on the other hand, refers to the situation in which a person simultaneously owes by some positive act,
loyalty to 2 or more states. While dual citizenship is involuntary, dual allegiance is the result of an individuals volition.
It is inimical to the national interest and shall be dealt with by law.

For citizens involved in such situation, by electing PH citizenship, such candidates at the same time forswear
allegiance to the other country which they are also citizens and thereby terminate their status as dual citizens. It may
be that, from the point of view of foreign state and of its laws, such an individual has not effectively renounced his
foreign citizenship and it is to be remembered that for people running for election it is important to explicitly renounce
foreign citizenship.

In the case at bar, the records show that Manzano was indeed a national both of PH and US however, COMELEC
en banc held that by participating in PH elections in 1992, 1995, and 1998, Manzano effectively renounced US
citizenship under American Law so that now, he is solely a PH national. Moreover, the filing of such certificate of
candidacy sufficed to renounce his US citizenship thus effectively removing any disqualification he might have as a
dual citizen.
There is, therefore, no merit in petitioner's contention that the oath of allegiance contained in private respondent's
certificate of candidacy is insufficient to constitute renunciation of his American citizenship. Equally without merit is
petitioner's contention that, to be effective, such renunciation should have been made upon private respondent
reaching the age of majority since no law requires the election of Philippine citizenship to be made upon majority age.
TAMPUS, MARY GRACE G. CONSTITUTIONAL LAW 1

Finally, much is made of the fact that private respondent admitted that he is registered as an American citizen in the
Bureau of Immigration and Deportation and that he holds an American passport which he used in his last travel to the
United States on April 22, 1997. There is no merit in this. Until the filing of his certificate of candidacy on March 21,
1998, he had dual citizenship. The acts attributed to him can be considered simply as the assertion of his American
nationality before the termination of his American citizenship.
TAMPUS, MARY GRACE G. CONSTITUTIONAL LAW 1

Jacot v. DAL and COMELEC

Jacot was a natural born citizen of the Philippines who became a naturalized citizen of the US. Later on, Jacot sought
to reacquire his PH citizenship under RA 9225 and filed his request for the administration of his Oath of Allegiance to
Republic of the Philippines with the PH Consulate General of Los Angeles, California. Los Angeles PCG issued an
order of Approval of Jacots request and on the same day, he took his Oath of Allegiance to Republic of Philippines
before Vice Consul Edward Yulo.

Six months after, Jacot filed his Certificate of Candidacy for the position of Vice Mayor of Catarman, Camiguin.
However, Dal filed a petition for Jacots disqualification arguing that Jacot failed to renounce his US citizenship as
required under Section 5 of RA 9225. In his answer, Jacot countered that his Oath of Allegiance to the Republic of
the Philippines made before Los Angeles PCG and oath contained in his Certificate of Candidacy operated as an
effective renunciation of his foreign citizenship.

In the meantime, National and Local Elections were held and Jacot garnered the highest number of votes for the
position of Vice Mayor.

COMELEC 2nd Division finally issued its Resolution disqualifying Jacot from running for the position of Vice Mayor of
Catarman, Camiguin for failure to make the requisite renunciation of his US citizenship. The COMELEC 2nd Division
explained that the reacquisition of PH Citizenship under RA 9225 does not automatically bestow upon any person the
privilege to run for any elective public office. It additionally ruled that the filing of a Certificate of Candidacy cannot be
considered as renunciation of foreign citizenship. Jacot filed a motion for reconsideration but COMELEC en banc
dismissed it.

Hence, the present petition.


ISSUE:
WON Jacot is disqualified from running as a candidate in the 2007 elections for his failure to make a personal and
sworn renunciation of his US citizenship

HELD:
Yes, he is disqualified from running as candidate for his failure to make a personal and sworn renunciation of his US
citizenship.
TAMPUS, MARY GRACE G. CONSTITUTIONAL LAW 1

Contrary to the assertions of Jacot, his oath of allegiance to the Republic of the Philippines made before Los Angeles
PCG and his Certificate of Candidacy do not substantially comply with the requirement of personal and sworn
renunciation of foreign citizenship because there are distinct requirements to be complied with for different purposes.

Section 3 of RA 9225 requires that natural-born citizens of PH, who are already naturalized citizens of a foreign
country, must take the following oath of allegiance to the Republic of the Philippines to reacquire or retain their PH
citizenship. By the oath dictated in the provision, Filipino swears allegiance to the PH but there is nothing on his
renunciation of foreign citizenship. The law categorically requires persons seeking elective public office who either
retained their PH citizenship or those who reacquired it, to make a personal and sworn renunciation of any and all
foreign citizenship before a public officer authorized to administer an oath simultaneous with or before the filing of
certificate of candidacy.

Hence, Section 5(2) of RA 9225 compels natural-born Filipinos, who have been naturalized as citizens of a foreign
country, but who reacquired or retained their PH citizenship to take the oath of allegiance under Section 3, RA 9225
and for those seeking elective public offices in the PH, to additionally execute a personal and sworn renunciation of
any and all foreign citizenship before an authorized public officer prior or simultaneous to the filing of their certificates
of candidacy, to qualify as candidates in PH elections.

To clarify, the oath of allegiance contained in the COC which is substantially similar to the one contained in Section 3
of RA 9225, does not constitute the personal and sworn renunciation sought under Section 5(2) of RA 9225. It bears
to emphasize that the said oath of allegiance is a general requirement for all those who wish to run as candidates in
the PH elections; while the renunciation of foreign citizenship is an additional requisite only for those who have
retained or reacquired PH citizenship under RA 9225 and who seek elective public posts, considering their special
circumstance of having more than one citizenship.
TAMPUS, MARY GRACE G. CONSTITUTIONAL LAW 1

Calilung v. Datumanong

FACTS
Calilung filed a petition against Secretary of Justice Datumanong, the official tasked to implement laws governing
citizenship praying that a writ of prohibition be issued to stop Datumanong from implementing RA 9225 An act
making the citizenship of PH citizens who acquire foreign citizenship permanent. Calilung further avers that RA 9225
is unconstitutional as it violates Section 5, Art.6 of the 1987 Constitution: dual allegiance of citizens is inimical to the
national interest and shall be dealt with by law. He also stated that RA 9225 cheapens Philippine citizenship, that
Secs.2 and 3 of RA 9225 allow dual allegiance and not dual citizenship and that this is very contrary to the PH
Constitution which states that dual allegiance is inimical to the national interest.
TAMPUS, MARY GRACE G. CONSTITUTIONAL LAW 1

ISSUE: WON RA 9225 is unconstitutional

HELD:

No, RA 9225 is not unconstitutional.

During the congressional deliberations, it is clear that the intent of the legislature in drafting RA 9225 is to do away
with the provision in CA 63 which takes away PH citizenship from natural-born Filipinos who become naturalized
citizens of other countries. What RA 9225 does is allow dual citizenship to natural-born Filipino citizens who have lost
PH citizenship by reason of their naturalization as citizens of a foreign country. On its face, it does not recognize dual
allegiance. By swearing to the supreme authority of Republic, the person implicitly renounces his foreign citizenship.
Plainly, from Sec. 3, RA 9225 stayed clear out of the problem of dual allegiance and shifted the burden of confronting
the issue of whether or not there is dual allegiance to the concerned foreign country. What happens to the other
citizenship was not made a concern of RA 9225.
TAMPUS, MARY GRACE G. CONSTITUTIONAL LAW 1

Magallona v. Ermita

FACTS:

In 1961 Congress passed RA 3046 demarcating the maritime baselines of the Philippines as an archipelagic state.
The law is framed after UNCLOS I which codifies the sovereign right of States parties over their territorial sea, but the
breadth was undetermined. Attempts to fill the void in negotiations done in UNCLOS II was futile therefore
domestically RA 3046 remain unchanged.

Then in 2009, Congress amended RA 3046 by enacting RA 9522. The change was prompted to make RA 3046
compliant with the terms of UNCLOS III. In UNCLOS III it provided already the water-land ratio, length, and contour
of baselines of archipelagic States. To comply with the requirements, RA 9522 shortened one baseline, optimized the
location of some basepoints around Philippine archipelago and classified adjacent territories as regime islands which
are Kalayaan Group of Islands and Bajo de Masinloc (Scarborough Shoal).

The enactment of RA 9522 prompted professors of law, law students and a legislator to assail the constitutionality of
the law that: (1) it reduces Philippine maritime territory and the reach of Philippine state sovereign power and (2) RA
9522 opens the countrys waters to maritime passage by all vessels and aircrafts, undermining Philippine sovereignty
and national sovereignty and damaging marine resources.

ISSUE/S:

-Preliminary

:WON Petitioners possess locus standi to bring the suit


:WON writs of certiorari and prohibition are the proper remedies to assail constitutionality of RA 9522
-Merits

:WON RA 9522 is unconstitutional

HELD:

-Preliminary

Petitioners possess locus standi.

Although their petition upon invoking their standing as legislators and taxpayers does not allocate them standing
since enactment of RA 9522 does not infringe the legislative prerogative nor was there any misuse of public funds
nonetheless, the Court recognizes their standing as citizens since the enactment of the law raises issues of national
significance necessitating urgent resolution.

As to the issue on writs of certiorari and prohibition as not the proper remedies to test the constitutionality of the
statute, it was held that upon the Courts exercise of its judicial review, it has been viewed as a tradition to see the
writs of certiorari and prohibition as proper remedial vehicles to test the constitutionality of statutes.
TAMPUS, MARY GRACE G. CONSTITUTIONAL LAW 1

-Merits

The Court finds RA 9522 constitutional and consistent with national interest and not violative of Art. 1 of Philippine
constitution.

RA 9522 merely demarcates the countrys maritime zones and continental shelves and does not delineate Philippine
territory furthermore, the law only abides the provisions in UNCLOS III. The Court importantly stressed that the
baseline laws are mere mechanisms for the UNCLOS III to precisely describe the delimitations. It serves as a notice
to the international family of states and it is in no way affecting or producing any effect like enlargement or diminution
of territories.

Moreover, the Court finds that the conversion of internal waters into archipelagic waters will not risk the Philippines
and that the State has sovereign powers that extends to the waters enclosed by the archipelagic baselines,
regardless of their depth or distance from the coast.

Lastly, contrary to the contention of petitioners, compliance to the UNCLOS III through RA 9522 will not expose
Philippine waters to nuclear and maritime pollution hazard for as a matter of fact, if Philippines did not comply with
the baselines law, it will devoid itself of internationally acceptable baselines from where the breadth of its maritime
zones and continental shelf is measured which will produce disaster: (1) open invitation to the seafaring powers to
freely enter and exploit the resources in the waters and submarine areas around the archipelago and (2) it will
weaken the countrys case in any international dispute over Philippine maritime space.
TAMPUS, MARY GRACE G. CONSTITUTIONAL LAW 1

Co Kim Cham v. Tan Keh

FACTS

Co Kim Cham had a pending civil case initiated during the Japanese occupation and that after the liberation of Manila,
Judge Arsenio Dizon refused to continue hearings on the said case saying that a proclamation issued by General
Douglas Macarthur had invalidated and nullified all judicial proceedings and judgments of the Court of Philippines
under Philippine Executive Commission and the Republic of the Philippines established during the Japanese military
occupation and that furthermore, lower courts have no jurisdiction to take cognizance of and continue judicial
proceedings in the courts of the defunct Republic of the Philippines in the absence of an enabling law granting such
authority. Additionally, he contends that the governments established in Philippines during Japanese occupation
were no de facto governments. Hence, present petition.

ISSUE:

1. WON under the rules of international law, judicial acts and proceedings of courts established in Philippines
were good and valid and remained as such even after the liberation or reoccupation of Philippines by US
and Filipino forces
2. If said judicial acts and proceedings have not been invalidated by said proclamation, WON the present
courts may continue those proceedings pending in said courts at the time Philippines was reoccupied and
liberated

HELD:

1. Yes. It is a legal truism in political and international law that all acts and proceedings of the legislative,
executive, and judicial departments of a de facto government are good and valid. The question to be
determined is whether or not the governments established in these Islands under the names of the
Philippine Executive Commission and Republic of the Philippines during the Japanese military occupation or
regime were de facto governments. If they were, the judicial acts and proceedings of those governments
remain good and valid even after the liberation or reoccupation of the Philippines by the American and
Filipino forces. There are several kinds of de facto governments. The first, or government de facto in a
TAMPUS, MARY GRACE G. CONSTITUTIONAL LAW 1

proper legal sense, is that government that gets possession and control of, or usurps, by force or by the
voice of the majority, the rightful legal governments and maintains itself against the will of the latter. The
second is that which is established and maintained by military forces who invade and occupy a territory of
the enemy in the course of war, and which is denominated a government of paramount force, and the third
is that established as an independent government by the inhabitants of a country who rise in insurrection
against the parent state.

According to that well-known principle in international law, the fact that a territory which has been occupied
by an enemy comes again into the power of its legitimate government of sovereignty, does not, except in a
very few cases, wipe out the effects of acts done by an invader, which for one reason or another it is within
his competence to do. Thus judicial acts done under his control, when they are not of a political complexion,
administrative acts so done, to the extent that they take effect during the continuance of his control, and the
various acts done during the same time by private persons under the sanction of municipal law, remain
good.

Therefore in this case judicial acts and proceedings of court are good and valid for only political laws are
abrogated and not that of judicial as well as administrative acts.

2. Yes. The proceedings in cases pending in the said court may continue without necessity of enacting a law
conferring jurisdiction upon them to continue said proceedings. The laws and courts of the Philippines did
not become laws and courts of Japan. Same Courts may continue exercising the same jurisdictions and
cases pending therein before the restoration of commonwealth until abolished and replaced by the said
government. \
TAMPUS, MARY GRACE G. CONSTITUTIONAL LAW 1

Government of Philippines v. Monte de Piedad

FACTS

An earthquake has happened in the Philippine islands on June 3, 1863 to which the Government of Spain
provided $400,000 as aid for the victims and was received by the Philippine Treasury. After the distribution of the
allotments to victims, $80,000 was left untouched and was then invested in Monte de Piedad bank which in turn
invested in jewelries.

On account of various petitions of the persons, and heirs of others to whom the above-mentioned allotments
were made by the central relief board for the payment of those amounts, the Philippine Islands to bring suit
against the Monte de Piedad a recover, through the Attorney General and in representation of the Government
of the Philippine Islands, the $80,000, together with interest, for the benefit of those persons or their heirs
appearing in the list of names published in the Official Gazette. Monte de Piedad bank argued that the Philippine
Government is not an affected party and has no right to institute a complaint and further provides that the
Government was not the intended beneficiary of the said amount.

ISSUE:

WON the Philippine Government is competent to file a complaint against the Monte de Piedad Bank

HELD:

Yes, the Philippine Government is competent to institute an action against Monte de Piedad Bank. The State as
a sovereign is the parens patriae of the people which means that the government being the protector of the
TAMPUS, MARY GRACE G. CONSTITUTIONAL LAW 1

rights of the people has the inherent supreme power to enforce such laws that will promote public interest. No
other party has been entrusted with such rights hence as parents of the people the government has the right to
take back the money intended for the people. Moreover, if the said loan was for ecclesiastical pious work, then
Spain would not exercise its civil capabilities.

SOVEREIGNTY

Tanada v. Angara

FACTS
Rizalino Navarro, then Secretary of the Department of Trade and Industry representing Government of the Republic
of PH, signed in Morocco, the results of the Uruguay Round of Multilateral Negotiations agreeing to the establishment
of the WTO.

In the same year, members of the PH Senate received letters from the President of the PH asking for its concurrence
in the ratification for the agreement establishing the World Trade Organization. Hence, Philippine Senate adopted
Resolution 97 concurring in the ratification by the President of the Philippines of the Agreement establishing World
Trade Organization.
TAMPUS, MARY GRACE G. CONSTITUTIONAL LAW 1

Because of the said concurrence done by the Congress, the present petition was filed stating among others that the
letter, spirit, and intent of the Constitution mandating economic nationalism is violated by the said Agreement
establishing WTO specifically constitutional provisions Sec. 19 of Article 2, Secs. 10 and 12 of Article 12 of the 1987
PH Constitution. They further allege that the national treatment and parity provisions of the WTO Agreement place
nationals and products of members of countries on the same footing as Filipinos and local products which is in
contravention of the Filipino first policy of the Constitution. Moreover, these provisions contravene constitutional
limitations on the role exports play in national development and negate the preferential treatment accorded to Filipino
labor, domestic materials and locally produced goods.

Respondents on the other hand replied that the provisions the petitioners are concerned of are not self-executing and
merely set out general policies and that these nationalistic portions of the Constitution invoked should not be read in
isolation but should be related to other relevant provisions of Article 12 and instead, the WTO Agreement contains
sufficient provisions protecting developing countries like the PH from the harshness of sudden trade liberalizations.

ISSUE:

1. WON the provisions of the WTO Agreement contravene Section 19, Article 2 and Section 10 and 12 of
Article 12 of the 1987 Constitution
2. WON the provisions of the WTO Agreement limit PH Sovereignty

HELD:

1. No, the provisions of the WTO Agreement does not contravene the provisions of the Constitution.

The provisions of the Constitution cited by the petitioners as basis for their cause of action are, by its very title,
declaration of principles and policies. They are used by the judiciary as aids or as guides in the exercise of its power
of judicial review, and by the legislature in its enactment of laws. As held in Kilosbayan v. Morato, the principles and
state policies enumerated in Article 2 and some sections of Article 12 are not self-executing provisions, the disregard
of which can give rise to a cause of action in courts. They do not really embody judicially enforceable constitutional
rights but guidelines for the legislation. It may be true however that in Manila Prince Hotel v. GSIS the Court held that
Sec.10, second paragraph of Article 12 of the 1987 Constitution is a mandatory and positive command which is
complete in itself not requiring any legislation to put it in operation. However, as the constitutional provision itself
states, it is enforceable only in regards to the grant of rights, privileges, and concessions covering national economy
and patrimony and not to every aspect of trade and commerce. It refers to exceptions rather than rules.

All told, while the Constitution indeed mandates a bias in favor of Filipino goods, services, labor and enterprises, at
the same time, it recognizes the need for business exchange with the rest of the world on the basis of equality and
reciprocity and limits protection of Filipino enterprises only against foreign competition and trade practices that are
unfair. In other words, Constitution did not intend to pursue an isolationist policy. It did not shut out foreign
investments, goods and services in the development of the PH economy. While the Constitution does not encourage
TAMPUS, MARY GRACE G. CONSTITUTIONAL LAW 1

unlimited entry of foreign goods, services and investments into the country, it does not prohibit them either. In fact, it
allows an exchange on the basis of equality and reciprocity, frowning only on foreign competition that is unfair.

2. No, the WTO provisions do not limit PH sovereignty.


While Sovereignty has traditionally been deemed absolute and all-encompassing on domestic level, it is however
subject to restrictions and limitations voluntarily agreed by the PH, expressly or impliedly, as a member of family of
nations. Unquestionably, Constitution does not envision a hermit-type isolation of the country from the rest of the
world. In its Declaration of Principles and State Policies, Constitution adopts the generally accepted principles of
international law as part of the law of the land, and adheres to policy of peace, equality, justice, freedom, cooperation
and amity, with all nations. By the doctrine of incorporation, the country is bound by generally accepted principles of
international law is pacta sunt servandainternational agreements must be performed in good faith. A treaty
engagement is not a mere moral obligation but creates legally binding obligation on the parties A state which has
contracted valid international obligations is bound to make in its legislations such modifications as may be necessary
to ensure fulfillment of obligations undertaken.

By their inherent nature, treaties really limit or restrict absoluteness of sovereignty. By States voluntary act, nations
may surrender some aspect of their state power in exchange for greater benefits granted by or derived from a
convention or pact. After all, states, like individuals, live with coequals, and in pursuit of mutuality covenanted
objectives and benefits, they also commonly agree to limit exercise of their absolute rights. Thus, treaties have been
used to record agreements between States concerning such widely diverse matters as, for example, the lease of
naval bases, sale or cession of territory, the termination of war, regulation of conduct of hostilities, formation of
alliances, regulation of commercial relations, settling of claims, laying down of rules governing conduct in peace and
establishment of international organizations. Sovereignty of a state therefore cannot in fact and in reality be
considered absolute. Certain restrictions enter into picture: 1) limitations imposed by the very nature of membership
in the family of nations and 2) limitations imposed by treaty stipulations. As aptly put by John Kennedy, Today, no
nation can build its destiny alone. The age of self-sufficient nationalism is over. The age of interdependence is here.

Thus, when Philippines joind UN as one of its 51 charter members, it consented to restrict its sovereignty rights under
the concept of sovereignty as auto-limitation.

In this sense, the Philippine Congress is restricted in its power to appropriate. It is compelled to appropriate funds
whether it agrees with such peace-keeping expenses or not. So too, under Article 105 of the said Charter, the UN
and its representatives enjoy diplomatic privileges and immunities, thereby limiting again the exercise of sovereignty
of members within their own territory. Another example: although "sovereign equality" and "domestic jurisdiction" of
all members are set forth as underlying principles in the UN Charter, such provisos are however subject to
enforcement measures decided by the Security Council for the maintenance of international peace and security
under Chapter VII of the Charter.

By agreeing to the WTO Agreement, the PH has effectively agreed to limit the exercise of its sovereign powers of
taxation, eminent domain and police power. The underlying consideration in this partial surrender of sovereignty is
the reciprocal commitment of other contracting states in granting the same privilege and immunities to the PH, its
officials and its citizens. The point in all of this is that, a portion of sovereignty may be waived without violating the
TAMPUS, MARY GRACE G. CONSTITUTIONAL LAW 1

Constitution, based on the rationale that the PH adopts generally accepted principles of international law as part of
the law of the land and adheres to the policy of cooperation and amity with all nations.
TAMPUS, MARY GRACE G. CONSTITUTIONAL LAW 1

Province of North Cotabato vs. GRP Peace Panel

FACTS

On August 5, 2008, Government of the Republic of the Philippines (GRP) and the MILF through their Chairpersons of
respective peace negotiating panels were scheduled to sign a memorandum of agreement on ancestral domains
aspect of GRP-MILF Tripoli Agreement on Peace of 2001 in Kuala Lumpur, Malaysia.

The MOA-AD was preceded by a long process of negotiations and concluding of prior agreements between MILF
and GRP. On July 18, 1997, GRP and MILF Peace Panels signed the agreement on General Cessation of Hostilities
and the following year, signed the General Framework of Agreement of Intent. However, it was already evident that
there was not going to be any smooth sailing in the GRP-MILF peace process.

A series of attacks by MILF as well as declaration of an all-out-war by the Government was declared. Eventually,
attacks ceased and negotiations resumed and several exploratory talks were held which eventually led to the crafting
of the draft of MOA-AD in its final form.

On July 27, 2008, Province of North Cotabato and Vice Governor Pinol filed a petition for mandamus and prohibition
with prayer for issuance of writ for preliminary injunction and temporary restraining order invoking right to information
on matters of public concern and of declaring MOA-AD to be unconstitutional hence, present petitions.

ISSUE/S:
1. WON the MOA-AD is inconsistent with the Constitution and the laws

HELD:
TAMPUS, MARY GRACE G. CONSTITUTIONAL LAW 1

Yes. The case invoked an international law on the concept of association to which Keitner and Reisman states that

An association is formed when two states of unequal power voluntarily establish durable links. In
the basic model, one state, the associate, delegates certain responsibilities to the other, the
principal, while maintaining its international status as a state. Free association represent a middle
ground between integration and independence.

In illustration of the theory of association, lets take the example between Marshall Islands and the US Government
wherein the Marshall Islands shall generally have the capacity to conduct foreign affairs in their own name and right,
such capacity extending to matters such as law of the sea, marine resources, trade, banking, postal, civil aviation and
cultural relations. In the event of attacks or threats against the Marshall Islands the US Government has authority and
obligation to defend them as if they were just part of the US territory.

Similarly in the present case, MOA-AD contains many provisions which are consistent with the international legal
concept of association specifically:
-Bangsamoro Juridical Entitys capacity to enter into economic and trade relations with foreign countries,
-the commitment of the Central Government to ensure Bangsamoro Juridical Entitys participation in meetings and
events in the ASEAN and specialized UN Agencies and,
-continuing responsibility of the Central Government over external defense.

Moreover, the BJEs right to participate in Philippine Official missions bearing on negotiation of border agreements,
environmental protection, and sharing of revenues pertaining to bodies of water adjacent or between islands forming
part of ancestral domain resembles the right of the government of Marshall Islands to be consulted by the US
Government.

This concept of association is NOT recognized under the present Constitution. It can then be said that the proposed
BJE is not merely an expanded version of ARMM. Indeed, BJE is a state in all but name as it meets the criteria of a
state laid down in Montevideo Convention namely a permanent population, a defined territory, a government, and a
capacity to enter into relations with other states.

Even assuming that BJE would not sever any portions of the Philippine territory, the spirit animating it which has
betrayed itself in use of the concept of association runs counter to the national sovereignty and territorial integrity of
the Republic.
Peralta v. Director of Prisons

FACTS
William Peralta, a member of the Metropolitan Constabulary of Manila charged with the supervision and control of the
production, procurement and distribution of goods and other necessaries was prosecuted for the crime of robbery as
defined and penalized by Section 2 of Act 65 of the National Assembly. He was found guilty and sentenced to life
imprisonment which he commenced to serve on August 21, 1944 by the Court of Special and Exclusive Criminal
Jurisdiction.
TAMPUS, MARY GRACE G. CONSTITUTIONAL LAW 1

A petition for habeas corpus was filed based on the contention that the Court of Special and Exclusive Criminal
Jurisdiction created by Ordinance No. 7 was a political instrumentality of the military forces of Japan and which is
repugnant to the aims of the Commonwealth of the Philippines for it does not afford fair trial and impairs the
constitutional rights of the accused.

ISSUE:
WON the creation of ordinance no. 7 is constitutional

HELD:
Yes, it is constitutional. There is no room for doubt to the validity of Ordinance No. 7 since the criminal jurisdiction
established by the invader is drawn entirely from the law martial as defined in the usages of nations. It is merely a
governmental agency. The sentence rendered, likewise, is good and valid since it was within the power and
competence of the belligerent occupant to promulgate Act No. 65. All judgments of political complexion of the courts
during Japanese regime ceased to be valid upon reoccupation of the Islands, as such, the sentence which convicted
the petitioner of a crime of a political complexion must be considered as having ceased to be valid.

Republic v. Villasor (G.R. No. L-30671, November 28, 1973)

FACTS
On June 24, 1969, Honorable Guillermo Villasor issued an order declaring that the decision be final and executory
directing the Sherriffs of Rizal Province. Pursuant to the said order, corresponding Alias Writ of Execution was issued.
On the strength of the said Alias Writ of Execution, the Provincial Sherriff of Rizal served notices of garnishment with
several banks especially on the monies due the Armed Forces of the Philippines in the form of deposits, sufficient to
TAMPUS, MARY GRACE G. CONSTITUTIONAL LAW 1

cover the amount mentioned in the said Writ of Execution. Philippine Veterans Bank received the same notice of
garnishment. It is important however, to note that, such funds are public funds duly appropriated and allocated for the
payment of pension of retirees, pay and allowances of military and civilian personnel and for maintenance and
operations of AFP.

Hence a petition is filed against Hon. Villasor as alleged to have acted in excess of his jurisdiction with grave abuse
of discretion in granting the issuance of an alias writ of execution against the properties of the AFP.

ISSUE:
WON the writ of execution issued by Hon. Villasor is valid

HELD:
No.
It is a fundamental postulate of constitutionalism flowing from the juristic concept of sovereignty that the state as well
as its government is immune from suit unless it gave its consent. It is readily understandable why it must be so. In the
classic formulation of Holmes: A sovereign is exempt from suit, not because of any formal conception or obsolete
theory, but on the logical and practical ground that there can be no legal right as against the authority that makes the
law on which the right depends.

The State may not be sued without its consent. A corollary, both dictated by logic and sound sense from such basic
concept is that public funds cannot be the object of a garnishment proceeding even if the consent to be sued had
been previously granted and the state liability adjudged.

"The universal rule that where the State gives its consent to be sued by private parties either by general or special
law, it may limit claimant's action `only up to the completion of proceedings anterior to the stage of execution' and
that the power of the Courts ends when the judgment is rendered, since government funds and properties may not be
seized under writs of execution or garnishment to satisfy such judgments, is based on obvious considerations of
public policy. Disbursements of public funds must be covered by the corresponding appropriation as required by law.
The functions and public services rendered by the State cannot be allowed to be paralyzed or disrupted by the
diversion of public funds from their legitimate and specific objects, as appropriated by law."

A rule, which has never been seriously questioned, is that money in the hands of public officers, although it may be
due government employees, is not liable to the creditors of these employees in the process of garnishment. One
reason is, that the State, by virtue of its sovereignty, may not be sued in its own courts except by express
authorization by the Legislature, and to subject its officers to garnishment would be to permit indirectly what is
prohibited directly. Another reason is that moneys sought to be garnished, as long as they remain in the hands of the
disbursing officer of the Government, belong to the latter, although the defendant in garnishment may be entitled to a
specific portion thereof. And still another reason which covers both of the foregoing is that every consideration of
public policy forbids it."
TAMPUS, MARY GRACE G. CONSTITUTIONAL LAW 1

Tan v. Director of Forestry (G.R. No.L-24548, Oct. 27, 1983)

FACTS:
Bureau of Forestry issued Notice No. 2087, advertising for public bidding a certain tract of public forest land situated
in Olongapo, Zambales, provided tenders were received. This public forest land, consisting of 6,420 hectares, is
located within the former U.S. Naval Reservation comprising 7,252 hectares of timberland, which was turned over by
the United States Government to the Philippine Government. Petitioner-appellant Wenceslao Vinzons Tan submitted
his application in due form after paying the necessary fees and posting the required bond therefor. Nine other
applicants submitted their offers before the deadline.

Thereafter, questions arose as to the wisdom of having the area declared as a forest reserve or allow the same to be
awarded to the most qualified bidder. On June 7, 1961, then President Carlos P. Garcia issued a directive to the
Director of the Bureau of Forestry to cancel the issuance of timber licenses which was sustained by the Secretary of
Agriculture and Natural Resources stating that it would be more beneficial to the public however allowed to proceed
with the announcement of results of the bidding for the said forest area which was then awarded to Vinzons Tan.

Against this award, other bidders protested and filed motions for reconsideration. On March 9, 1964, The Secretary
of ANR declared Tans OTL null and void (but the same was not granted to Ravago). Petitioner-appellant moved for a
reconsideration of the order, but the Secretary of Agriculture and Natural Resources denied the motion.

ISSUE:
1. WON Tans action will prosper

HELD:
No, as the action is a suit against the State.
The Director of Forestry in revoking the license only acted within the scope of their authority. Accordingly, "the rule
establishing State exemption from suits may not be circumvented by directing the action against the officers of the
State instead of against the State itself. In such cases the State's immunity may be validly invoked against the action
as long as it can be shown that the suit really affects the property, rights, or interests of the State and not merely
those of the officer nominally made party defendant".

Both the Secretary of Agriculture and Natural Resources and the Director of Forestry acted in their capacity as
officers of the State, representatives of the sovereign authority discharging governmental powers. A private individual
cannot issue a timber license.
Consequently, a favorable judgment for the petitioner-appellant would result in the government losing a substantial
part of its timber resources. This being the case, petitioner-appellant's action cannot prosper unless the State gives
its consent to be sued.
TAMPUS, MARY GRACE G. CONSTITUTIONAL LAW 1

Republic v. Feliciano (G.R. No. 70853, March, 1987)


FACTS
Feliciano bought a parcel of land from Victor Gardiola through a contract of Sale evidenced by an informacion
possesoria which upon purchase he immediately took actual possession of the said property and introduced various
improvements therein.
Many years later however, the said parcel of land was included under the coverage of President Ramon
Magsaysays program NARRA for settlement purposes because of this, Feliciano filed a complaint in CFI of
Camarines Sur against Republic of the Philippines represented by Land Authority that the said parcel of land should
be excluded from the program and that he be declared the rightful and true owner of the property in question.

RTC rendered a decision that the said parcel of land be excluded therefrom in the NARRA Settlement reservation
which a motion to intervene was filed by the settlers alleging that they have already been in possession of the land
and has been doing so for more than 20 years already.

Hence this petition.

ISSUE:
WON the action may prosper
HELD:
No, the action will not prosper. The doctrine of non-suability of the State has proper application in this case. The
plaintiff has impleaded the Republic of the Philippines as defendant in an action for recovery of ownership and
possession of a parcel of land, bringing the State to court just like any private person who is claimed to be usurping a
piece of property. A suit for the recovery of property is not an action in rem, but an action in personam. It is an action
directed against a specific party or parties, and any judgment therein binds only such party or parties. The complaint
filed by plaintiff, the private respondent herein, is directed against the Republic of the Philippines, represented by the
Land Authority, a governmental agency.

By its caption and its allegation and prayer, the complaint is clearly a suit against the State, which under settled
jurisprudence is not permitted, except upon a showing that the State has consented to be sued, either expressly or
by implication through the use of statutory language too plain to be misinterpreted. There is no such showing in the
TAMPUS, MARY GRACE G. CONSTITUTIONAL LAW 1

instant case. Worse, the complaint itself fails to allege the existence of such consent. This is a fatal defect, and on
this basis alone, the complaint should have been dismissed.

The failure of the petitioner to assert the defense of immunity from suit when the case was tried before the court a
quo, as alleged by private respondent, is not fatal. It is now settled that such defense "may be invoked by the courts
sua sponte at any stage of the proceedings."

Private respondent contends that the consent of petitioner may be read from the Proclamation itself, when it
established the reservation "subject to private rights, if any there be." We do not agree. No such consent can be
drawn from the language of the Proclamation. The exclusion of existing private rights from the reservation
established by Proclamation No. 90 can not be construed as a waiver of the immunity of the State from suit. Waiver
of immunity, being a derogation of sovereignty, will not be inferred lightly, but must be construed in strictissimi juris.
Moreover, the Proclamation is not a legislative act. The consent of the State to be sued must emanate from statutory
authority. Waiver of State immunity can only be made by an act of the legislative body.

Department of Agriculture v. NLRC


FACTS
DA and Sultan Security Agency entered into a contract for security services to be provided to the said government
entity. Save for the increase in monthly rate of the guards, same terms and conditions were made to apply to another
contract between the same parties.

A year after, several guards of the Sultan Security Agency filed a complaint for underpayment of wages, non-
payment of 13th month pay, uniform allowances, night shift differential pay, holiday pay and overtime pay, as well as
for damages against DA and Sultan Security Agency.
The Executive Labor Arbiter rendered a decision finding herein petitioner jointly and severally liable with sultan
Security Agency for the payment of the money claims, aggregating P266,483.91, of the complainant security guards.
The petitioner and Sultan Security Agency did not appeal the decision of the Labor Arbiter. Thus, the decision
became final and executory. The Labor Arbiter issued a writ of execution, commanding the City Sheriff to enforce and
execute the judgment against the property of the two respondents.

A petition for injunction, prohibition and mandamus, with prayer for preliminary writ of injunction, was filed by the
petitioner with the National Labor Relations Commission ("NLRC"), Cagayan de Oro, alleging, inter alia, that the writ
issued was effected without the Labor Arbiter having duly acquired jurisdiction over the petitioner, and that, therefore,
TAMPUS, MARY GRACE G. CONSTITUTIONAL LAW 1

the decision of the Labor Arbiter was null and void and all actions pursuant thereto should be deemed equally invalid
and of no legal effect.

However, NLRC promulgated a resolution dismissing the injuction filed by DA. Hence, in a petition for certiorari DA
charges NLRC with grave abuse of discretion for refusing to quash the writ of execution and went on even if it were
to assume that NLRC has jurisdiction over a money claim against the Department, NLRC has disregarded the
cardinal rule on non-suability of the State.

ISSUE:
WON DA may invoke the doctrine of non-suability of the State as way of defense for the money claims involved in
the contract

HELD:
Yes.
The basic postulate enshrined in the constitution that "(t)he State may not be sued without its consent," reflects
nothing less than a recognition of the sovereign character of the State and an express affirmation of the unwritten
rule effectively insulating it from the jurisdiction of courts. It is based on the very essence of sovereignty. As has been
aptly observed, by Justice Holmes, a sovereign is exempt from suit, not because of any formal conception or
obsolete theory, but on the logical and practical ground that there can be no legal right as against the authority that
makes the law on which the right depends.

The rule, in any case, is not really absolute for it does not say that the state may not be sued under any circumstance.
On the contrary, as correctly phrased, the doctrine only conveys, "the state may not be sued without its consent;" its
clear import then is that the State may at times be sued. The States' consent may be given either expressly or
impliedly. Express consent may be made through a general law or a special law. In this jurisdiction, the general law
waiving the immunity of the state from suit is found in Act No. 3083, where the Philippine government "consents and
submits to be sued upon any money claim involving liability arising from contract, express or implied, which could
serve as a basis of civil action between private parties." Implied consent, on the other hand, is conceded when the
State itself commences litigation, thus opening itself to a counterclaim or when it enters into a contract.

Applying in the present situation, the government is deemed to have descended to the level of the other contracting
party and to have divested its sovereign immunity as not all contracts entered into by the Government operates as
waiver of its non-suability, distinction must still be made between one which is executed in the exercise of its
sovereign functions and another which is done in its proprietary capacity.

In the instant case, the Department of Agriculture has not pretended to have assumed a capacity apart from its being
a governmental entity when it entered into the questioned contract; nor that it could have, in fact, performed any act
proprietary in character. But, be that as it may, the claims of private respondents, i.e., for underpayment of wages,
holiday pay, overtime pay and similar other items, arising from the Contract for Security Services, clearly constitute
money claims. When the State gives its consent to be sued, it does not thereby necessarily consent to an
unrestrained execution against it. Tersely put, when the State waives its immunity, all it does, in effect, is to give the
other party an opportunity to prove, if it can, that the State has a liability.
TAMPUS, MARY GRACE G. CONSTITUTIONAL LAW 1
TAMPUS, MARY GRACE G. CONSTITUTIONAL LAW 1

Sanders v. Veridiano

FACTS
Sanders was, at the time the incident in question occurred, the special services director of the U.S. Naval Station
(NAVSTA) in Olongapo City. Petitioner Moreau was the commanding officer of the Subic Naval Base, which includes
the said station. Private respondent Rossi is an American citizen with permanent residence in the Philippines, 3 as so
was private respondent Wyer.

Rossi and Wyer were advised that their employment had been converted from permanent full-time to permanent part-
time their reaction therein was to protest and institute grievance proceedings conformably to the pertinent rules and
regulations of US Department of defense. The result was a recommendation from the hearing officer who conducted
the proceedings for the reinstatement of the private respondents to permanent full-time status plus backwages.

In a letter addressed to petitioner Moreau Sanders disagreed with the hearing officer's report and asked for the
rejection of the abovestated recommendation. The letter contained the statements that: a) "Mr. Rossi tends to
alienate most co-workers and supervisors;" b) "Messrs. Rossi and Wyers have proven, according to their immediate
supervisors, to be difficult employees to supervise;" and c) "even though the grievants were under oath not to discuss
the case with anyone, placed the records in public places where others not involved in the case could hear.

Hence, Rossi and Wyer filed in CFI a complaint for damages and claimed that the letters were libelous imputations
that had exposed them to ridicule and caused them mental anguish and that the prejudgment of grievance
proceedings was an invasion of their personal and proprietary rights. The private respondents made it clear that the
petitioners were being sued in their private or personal capacity. However, in a motion to dismiss filed under a special
appearance, the petitioners argued that the acts complained of were performed by them in the discharge of their
official duties and that, consequently, the court had no jurisdiction over them under the doctrine of state immunity.

ISSUE: WON Sanders and Moreau acted officially when they did the acts for which Rossi and Wyer have sued them
for

HELD:
Yes, Sanders and Moreau acted in their official capacity.
It is stressed at the outset that the mere allegation that a government functionary is being sued in his personal
capacity will not automatically remove him from the protection of the law of public officers and, if appropriate, the
doctrine of state immunity. By the same token, the mere invocation of official character will not suffice to insulate him
from suability and liability for an act imputed to him as a personal tort committed without or in excess of his authority.
These well-settled principles are applicable not only to the officers of the local state but also where the person sued
in its courts pertains to the government of a foreign state, as in this case.
TAMPUS, MARY GRACE G. CONSTITUTIONAL LAW 1

It is abundantly clear in the present case that the acts for which the petitioners are being called to account were
performed by them in the discharge of their official duties. Sanders, as director of the special services department of
NAVSTA, undoubtedly had supervision over its personnel, including the private respondents, and had a hand in their
employment, work assignments, discipline, dismissal and other related matters. It is not disputed that the letter he
had written was in fact a reply to a request from his superior, the other petitioner, for more information regarding the
case of the private respondents. Moreover, even in the absence of such request, he still was within his rights in
reacting to the hearing officer's criticism in effect a direct attack against him that Special Services was
practicing "an autocratic form of supervision."

Given the official character of the above-described letters, we have to conclude that the petitioners were, legally
speaking, being sued as officers of the United States government. As they have acted on behalf of that government,
and within the scope of their authority, it is that government, and not the petitioners personally, that is responsible for
their acts. Assuming that the trial can proceed and it is proved that the claimants have a right to the payment of
damages, such award will have to be satisfied not by the petitioners in their personal capacities but by the United
States government as their principal. This will require that government to perform an affirmative act to satisfy the
judgment, viz., the appropriation of the necessary amount to cover the damages awarded, thus making the action a
suit against that government without its consent.

All this is not to say that in no case may a public officer be sued as such without the previous consent of the state. To
be sure, there are a number of well-recognized exceptions. It is clear that a public officer may be sued as such to
compel him to do an act required by law, as where, say, a register of deeds refuses to record a deed of sale; or to
restrain a Cabinet member, for example, from enforcing a law claimed to be unconstitutional; or to compel the
national treasurer to pay damages from an already appropriated assurance fund; or the commissioner of internal
revenue to refund tax overpayments from a fund already available for the purpose; or, in general, to secure a
judgment that the officer impleaded may satisfy by himself without the government itself having to do a positive act to
assist him. We have also held that where the government itself has violated its own laws, the aggrieved party may
directly implead the government even without first filing his claim with the Commission on Audit as normally required,
as the doctrine of state immunity "cannot be used as an instrument for perpetrating an injustice."

The case at bar, to repeat, comes under the rule and not under any of the recognized exceptions. The government of
the United States has not given its consent to be sued for the official acts of the petitioners, who cannot satisfy any
judgment that may be rendered against them. As it is the American government itself that will have to perform the
affirmative act of appropriating the amount that may be adjudged for the private respondents, the complaint must be
dismissed for lack of jurisdiction.
TAMPUS, MARY GRACE G. CONSTITUTIONAL LAW 1

Mobil PH Exploration v. Customs Arrastre Service and Bureau of Customs

FACTS

Four cases of rotary drill parts shipped from abroad via S.S. Leoville, consigned to Mobil Philippines Exploration,
Inc., Manila arrived sometime in November 1962 at the Port of Manila and was released to the protection of the
Customs Arrastre Service (CAS), operations, the unit of the Bureau of Customs that handles such arrastre operations.
However, instead of the expected four cases, CAS was only able to deliver three. The plaintiff, Mobil Phils., filed a
suit in the CFI of Manila against the two entities, to recover the value of the missing case at 18,493.37 pesos plus
damages. Meanwhile, the defendants filed a motion to dismiss the complaint claiming that both entities were not
suable. The appellant now contends that not all government entities are immune from suit; that defendant Bureau of
Customs as operator of the arrastre service at the Port, is discharging proprietary functions and as such, can be sued
by private individuals.

ISSUE: Whether or not the defendants can use State Immunity as a defense.

HELD:
TAMPUS, MARY GRACE G. CONSTITUTIONAL LAW 1

Yes. The court affirmed the order of dismissal for appeal, with costs against the appellant. The defendants were
performing a function proprietary in nature, however, as the Bureau of Customs is part of the Department of Finance,
its personality is as part of the national government. Its primary function is assessing and collecting revenues from
imported articles and all other tariff and customs duties, a governmental function. Although the function of arrastre
service is deemed proprietary, it is a necessary unit of the primary and governmental function of the Bureau of
Customs, and engaging in the same does not necessarily render said Bureau liable to suit. Otherwise, it could not
perform one of its governmental functions if it is vulnerable to suit. Therefore, the defendants at the case at bar can
use the doctrine of Sovereign or State Immunity as a defense.

Kuroda v. Jalandoni

Facts:
Petitioner Sheginori Kuroda was the former Lt. General of the Japanese Army and commanding general of the
Japanese forces during the occupation (WWII) in the country. He was tried before the Philippine Military Commission
TAMPUS, MARY GRACE G. CONSTITUTIONAL LAW 1

for War Crimes and other atrocities committed against military and civilians. The military commission was establish
under Executive Order 68.

Petitioner assails the validity of EO 68 arguing it is unconstitutional and hence the military commission did not have
the jurisdiction to try him on the following grounds:
- that the Philippines is not a signatory to the Hague Convention (War Crimes)

Petitioner likewise assails that the US is not a party of interest in the case hence the 2 US prosecutors cannot
practice law in the Philippines.

Issue: Whether or not EO 68 is constitutional thus the military tribunal jurisdiction is valid

HELD:

EO 68 is constitutional hence the tribunal has jurisdiction to try Kuroda. EO 68 was enacted by the President and
was in accordance with Sec. 3, Art. 2 of Constitution which renounces war as an instrument of national policy. Hence
it is in accordance with generally accepted principles of international law including the Hague Convention and
Geneva Convention, and other international jurisprudence established by the UN, including the principle that all
persons (military or civilian) guilty of plan, preparing, waging a war of aggression and other offenses in violation of
laws and customs of war. The Philippines may not be a signatory to the 2 conventions at that time but the rules and
regulations of both are wholly based on the generally accepted principles of international law. They were accepted
even by the 2 belligerent nations (US and Japan)

As to the participation of the 2 US prosecutors in the case, the US is a party of interest because its country and
people have greatly aggrieved by the crimes which petitioner was being charged of.

Moreover, the Phil. Military Commission is a special military tribunal and rules as to parties and representation are
not governed by the rules of court but the provision of this special law.
TAMPUS, MARY GRACE G. CONSTITUTIONAL LAW 1

Philip Morris v. CA

FACTS:

This is a petition for review under Rule 45 of the Rules of Court, to seek the reversal and setting aside of the
following issuances of the Court of Appeals (CA).

Philip Morris, Inc. and two other petitioners are ascribing whimsical exercise of the faculty conferred upon
magistrates by Section 6, Rule 58 of the Revised Rules of Court when respondent Court of Appeals lifted the writ of
preliminary injunction it earlier had issued against Fortune Tobacco Corporation, from manufacturing and selling
MARK cigarettes in the local market. Banking on the thesis that petitioners respective symbols MARK VII, MARK
TEN, and MARK, also for cigarettes, must be protected against unauthorized appropriation.

All petitioners are not doing business in the Philippines but are suing on an isolated transaction, They Invoked
provisions of the Paris Convention for the Protection of Industrial and Intellectual Property. As corporate nationals of
member-countries of the Paris Union, they can sue before Philippine courts for infringement of trademarks, or for
unfair competition, without need of obtaining registration or a license to do business in the Philippines, and without
necessity of actually doing business in the Philippines.

Philip Morris and its subsidiaries filed the complaint for infringement and damages against Fortune Tobacco before
the Pasig Regional Trial Court (RTC) for manufacturing and selling cigarettes bearing the trademark Mark which is
identical and confusingly similar to Philip Morris trademarks. The said act was dismissed. Hence, this petition.

ISSUE:

Whether or not there has been an invasion of plaintiffs right of property to such trademark or trade name.

HELD:

Following universal acquiescence and comity, our municipal law on trademarks regarding the requirement of actual
use in the Philippines must subordinate an international agreement inasmuch as the apparent clash is being decided
TAMPUS, MARY GRACE G. CONSTITUTIONAL LAW 1

by a municipal tribunal. Withal, the fact that international law has been made part of the law of the land does not by
any means imply the primacy of international law over national law in the municipal sphere. Under the doctrine of
incorporation as applied in most countries, rules of international law are given a standing equal, not superior, to
national legislative enactments.

Sec. of Justice v. Lantion

FACTS

On 1977, President Marcos issued PD 1069 Prescribing the Procedure for the Extradition of Persons who have
committed Crimes in a Foreign Country. The Decree was founded on the doctrine of incorporation under the
Constitution with the mutual concern of suppression of crime both in the State where it was committed and the State
where criminal have escaped.

On 1994 as well, Secretary Franklin Drilon representing Government of the Republic of Philippines signed the RP-US
Extradition Treaty.

Then on 1999, Department of Justice received from Department of Foreign Affairs US Note Verbale No. 0522
containing a request for the extradition of Mark Jimenez to US attached therein were the Grand Jury Indictment, the
warrant of arrest and other supporting documents for said extradition. On the same day, Government of US issued
Department Order 249 designating and authorizing panel of attorneys to take charge and handle the case.

Mark Jimenez then wrote a letter to Justice Secretary requesting copies of the official extradition request from the
U.S Government and that he be given ample time to comment on the request after he shall have received copies of
the requested papers but the petitioner denied the request for the consistency of Article 7 of the RP-US Extradition
Treaty stated in Article 7 that the Philippine Government must present the interests of the United States in any
proceedings arising out of a request for extradition.

ISSUE: Whether or not to uphold a citizens basic due process rights or the governments ironclad duties under a
treaty.
TAMPUS, MARY GRACE G. CONSTITUTIONAL LAW 1

RULING: Petition dismissed.


The human rights of person, whether citizen or alien , and the rights of the accused guaranteed in our Constitution
should take precedence over treaty rights claimed by a contracting state. The duties of the government to the
individual deserve preferential consideration when they collide with its treaty obligations to the government of another
state. This is so although we recognize treaties as a source of binding obligations under generally accepted
principles of international law incorporated in our Constitution as part of the law of the land.
The doctrine of incorporation is applied whenever municipal tribunals are confronted with situation in which there
appears to be a conflict between a rule of international law and the provision of the constitution or statute of the local
state.

Petitioner (Secretary of Justice) is ordered to furnish Mark Jimenez copies of the extradition request and its
supporting papers, and to grant him (Mark Jimenez) a reasonable period within which to file his comment with
supporting evidence. Under the Doctrine of Incorporation, rules of international law form part of the law of the land
and no further legislative action is needed to make such rules applicable in the domestic sphere. The doctrine of
incorporation is applied whenever municipal tribunals are confronted with situations in which there appears to be a
conflict between a rule of international law and the provisions of the constitution or statute of the local state. Efforts
should first be exerted to harmonize them, so as to give effect to both since it is to be presumed that municipal law
was enacted with proper regard for the generally accepted principles of international law in observance of the
incorporation clause in the above cited constitutional provision.

In a situation, however, where the conflict is irreconcilable and a choice has to be made between a rule of
international law and a municipal law, jurisprudence dictates that municipal law should be upheld by the municipal
courts, for the reason that such courts are organs of municipal law and are accordingly bound by it in all
circumstances. The fact that international law has been made part of the law of the land does not pertain to or imply
the primacy of international law over national or municipal law in the municipal sphere. The doctrine of incorporation,
as applied in most countries, decrees that rules of international law are given equal standing with, but are not
superior to, national legislative enactments. Accordingly, the principle lex posterior derogate priori takes effect a
treaty may repeal a statute and a statute may repeal a treaty. In states where the Constitution is the highest law of
the land, such as the Republic of the Philippines, both statutes and treaties may be invalidated if they are in conflict
with the constitution.
TAMPUS, MARY GRACE G. CONSTITUTIONAL LAW 1

Government of US v. Purganan

FACTS
Finding no more legal obstacle, the Government of the United States of America, represented by the Philippine DOJ,
filed with the RTC on 18 May 2001, the appropriate Petition for Extradition. The Petition alleged, inter alia, that
Jimenez was the subject of an arrest warrant issued by the United States District Court for the Southern District of
Florida on 15 April 1999.

Before the RTC could act on the Petition, Jimenez filed before it an Urgent Manifestation/Ex-Parte Motion, which
prayed that Jimenezs application for an arrest warrant be set for hearing. In its 23 May 2001 Order, the RTC granted
the Motion of Jimenez and set the case for hearing on 5 June 2001. In that hearing, Jimenez manifested its
TAMPUS, MARY GRACE G. CONSTITUTIONAL LAW 1

reservations on the procedure adopted by the trial court allowing the accused in an extradition case to be heard prior
to the issuance of a warrant of arrest.

After the hearing, the court a quo required the parties to submit their respective memoranda. In his Memorandum,
Jimenez sought an alternative prayer: that in case a warrant should issue, he be allowed to post bail in the amount of
P100,000.

The alternative prayer of Jimenez was also set for hearing on 15 June 2001. Thereafter, the court below issued its 3
July 2001 Order, directing the issuance of warrant for his arrest and fixing bail for his temporary liberty at P1 million in
cash. After he had surrendered his passport and posted the required cash bond, Jimenez was granted provisional
liberty via the challenged Order dated 4 July 2001. Hence, this petition.

ISSUES:
1. Whether Jimenez is entitled to notice and hearing before a warrant for his arrest can be issued
2. Whether he is entitled to bail and to provisional liberty while the extradition proceedings are pending
Held:
1. No.
To determine probable cause for the issuance of arrest warrants, the Constitution itself requires only the examination
under oath or affirmation of complainants and the witnesses they may produce. There is no requirement to
notify and hear the accused before the issuance of warrants of arrest.

At most, in cases of clear insufficiency of evidence on record, judges merely further


examine complainants and their witnesses. In the present case, validating the act of respondent judge and instituting
the practice of hearing the accused and his witnesses at this early stage would be discordant with the rationale for
the entire system. If the accused were allowed to be heard and necessarily to present evidence during the prima
facie determination for the issuance of a warrant of arrest, what would stop him from presenting his entire plethora of
defenses at this stage if he so desires in his effort to negate a prima facie finding? Such a procedure could
convert the determination of a prima facie case into a full-blown trial of the entire proceedings and possibly make trial
of the main case superfluous. This scenario is also anathema to the summary nature of extraditions.

Upon receipt of a petition for extradition and its supporting documents, the judge must study them and make, as soon
as possible, a prima facie finding whether (a) they are sufficient in form and substance, (b) they show compliance
with the Extradition Treaty and Law, and (c) the person sought is extraditable. At his discretion, the judge may
require the submission of further documentation or may personally examine the affiants and witnesses of the
petitioner. If, in spite of this study and examination, no prima facie finding is possible, the petition may be dismissed
at the discretion of the judge.
On the other hand, if the presence of a prima facie case is determined, then the magistrate must immediately issue a
warrant for the arrest of the extraditee, who is at the same time summoned to answer the petition and to appear at
scheduled summary hearings. Prior to the issuance of the warrant, the judge must not inform or notify the potential
extraditee of the pendency of the petition, lest the latter be given the opportunity to escape and frustrate the
proceedings. In our opinion, the foregoing procedure will best serve the ends of justice in extradition cases.
TAMPUS, MARY GRACE G. CONSTITUTIONAL LAW 1

2. No.
Extradition cases are different from ordinary criminal proceedings. The constitutional right to bail flows from the
presumption of innocence in favor of every accused who should not be subjected to the loss of freedom as thereafter
he would be entitled to acquittal, unless his guilt be proved beyond reasonable doubt.It follows that the constitutional
provision on bail will not apply to a case like extradition, where the presumption of innocence is not at issue.

Respondent Jimenez cites the foreign case Parettiin arguing that, constitutionally, [n]o one shall be deprived of x x
x liberty x x x without due process of law.
Contrary to his contention, his detention prior to the conclusion of the extradition proceedings does not amount to a
violation of his right to due process. We iterate the familiar doctrine that the essence of due process is the opportunity
to be heard but, at the same time, point out that the doctrine does not always call for a prior opportunity to be heard.
Where the circumstances such as those present in an extradition case call for it, a subsequent opportunity to
be heard is enough. In the present case, respondent will be given full opportunity to be heard subsequently, when the
extradition court hears the Petition for Extradition. Hence, there is no violation of his right to due process and
fundamental fairness.

Ichong v. Hernandez
TAMPUS, MARY GRACE G. CONSTITUTIONAL LAW 1

FACTS

Congress passed a legislation which is RA 1180 entitled An Act to Regulate the Retail Business nationalizing the
retail business. Its purpose was to prevent persons who are not citizens of the Phil. from having a stranglehold upon
the peoples economic life.

a prohibition against aliens and against associations, partnerships, or corporations the capital of which are
not wholly owned by Filipinos, from engaging directly or indirectly in the retail trade
aliens actually engaged in the retail business on May 15, 1954 are allowed to continue their business,
unless their licenses are forfeited in accordance with law, until their death or voluntary retirement. In case of
juridical persons, ten years after the approval of the Act or until the expiration of term.

Citizens and juridical entities of the United States were exempted from this Act.

provision for the forfeiture of licenses to engage in the retail business for violation of the laws on
nationalization, economic control weights and measures and labor and other laws relating to trade,
commerce and industry.
provision against the establishment or opening by aliens actually engaged in the retail business of additional
stores or branches of retail business

Lao Ichong, in his own behalf and behalf of other alien residents, corporations and partnerships affected by the Act,
filed an action to declare it unconstitutional for the ff: reasons:

it denies to alien residents the equal protection of the laws and deprives them of their liberty and property
without due process
the subject of the Act is not expressed in the title
the Act violates international and treaty obligations
the provisions of the Act against the transmission by aliens of their retail business thru hereditary
succession

ISSUE: WON RA 1180 violates the equal protection clause

HELD:

The law is a valid exercise of police power and it does not deny the aliens the equal protection of the laws. There are
real and actual, positive and fundamental differences between an alien and a citizen, which fully justify the legislative
classification adopted.

The equal protection clause does not demand absolute equality among residents. It merely requires that all persons
shall be treated alike, under like circumstances and conditions both as to privileges conferred and liabilities enforced.

The classification is actual, real and reasonable, and all persons of one class are treated alike.

The difference in status between citizens and aliens constitutes a basis for reasonable classification in the exercise of
police power.
TAMPUS, MARY GRACE G. CONSTITUTIONAL LAW 1

Official statistics point out to the ever-increasing dominance and control by alien of the retail trade. It is this
domination and control that is the legislatures target in the enactment of the Act.

The mere fact of alienage is the root cause of the distinction between the alien and the national as a trader. The alien
is naturally lacking in that spirit of loyalty and enthusiasm for the Phil. where he temporarily stays and makes his
living. The alien owes no allegiance or loyalty to the State, and the State cannot rely on him/her in times of crisis or
emergency.

While the citizen holds his life, his person and his property subject to the needs of the country, the alien may become
the potential enemy of the State.

The alien retailer has shown such utter disregard for his customers and the people on whom he makes his profit.
Through the illegitimate use of pernicious designs and practices, the alien now enjoys a monopolistic control on the
nations economy endangering the national security in times of crisis and emergency.
TAMPUS, MARY GRACE G. CONSTITUTIONAL LAW 1

Gonzales v. Hechanova

FACTS

During the term of President Diosdado Macapagal, he entered into an agreement with Vietnam and Burma for the
importation of rice without complying with the requisite of securing a certification from National Economic Council
showing that there is a shortage in cereals or rice. Hence, the then Executive Secretary Hechanova authorized the
importation of 67,000 tons of rice from abroad to the detriment of the local rice planters. Romano Gonzales, the
petitioner, assailed the executive agreements stating that the authorization of the importation of the rice violates RA
3452 which prohibits the importation of rice and corn by the Rice and Corn Administration or any government agency.

ISSUE:

Whether or not RA 3452 prevails over the 2 executive agreements entered into by Macapagal

HELD:

Yes. Under the Constitution the main function of the Executive is to enforce laws enacted by the Congress. The
former may not interfere in the performance of the legislative powers of the Congress except in exercise of his veto
power. He may not defeat legislative enactments that have acquired the status of laws, by indirectly repealing the
same through an executive agreement providing for the performance of the very act prohibited by said laws. In the
event of conflict between a treaty and a statute, the one which is latest in point of time shall prevail, is not applicable
to the case at bar. No such justification can be given as regards executive agreements not authorized by previous
legislation, without completely upsetting the principle of separation of powers and the system of checks and balances
which are fundamental in our constitutional set up.

As regards the question whether an executive or an international agreement may be invalidated by our courts, suffice
it to say that the Constitution of the Philippines has clearly settled it in the affirmative, by providing that SC may not
be deprived of its jurisdiction to review, revise, reverse, modify, or affirm on appeal, certiorari, or writ of error, as the
law or the rules of court may provide, final judgments and decrees of inferior courts in all cases in which the
constitutionality or validity of any treaty, law, ordinance, or executive order or regulation is in question. In other words,
our Constitution authorizes the nullification of a treaty, not only when it conflicts with the fundamental law, but, also,
when it runs counter to an act of Congress.
TAMPUS, MARY GRACE G. CONSTITUTIONAL LAW 1

Ebralinag v. Division Superintendent of Schools Cebu

FACTS

DECS Regional Office in Cebu received complaints about teachers and pupils belonging to the Jehovahs
Witness, and enrolled in various public and private schools, which refused to sing the Phil. National
Anthem, salute the flag and recite the patriotic pledge.

Division Superintendent of schools, Susana B. Cabahug of the Cebu Division of DECS and her Assistant
issued Division Memorandum No. 108, dated Nov. 17, 1989, directing District Supervisors, High School
Principals and Heads of Private Educational institutions to remove from service, after due process,
teachers and school employees, and to deprive the students and pupils from the benefit of public
education, if they do not participate in daily flag ceremony and doesnt obey flag salute rule.

Members of the Jehovahs Witness sect find such memorandum to be contrary to their religious belief and
choose not to obey. Despite a number of appropriate persuasions made by the Cebu officials to let them
obey the directives, still they opted to follow their conviction to their belief. As a result, an order was
issued by the district supervisor of Daan Bantayan District of Cebu, dated July 24, 1990, ordering the
dropping from the list in the school register of all Jehovahs Witness teachers and pupils from Grade 1 to
Grade 6 who opted to follow their belief which is against the Flag Salute Law, however, given a chance to
be re-accepted if they change their mind.

Some Jehovahs Witness members appealed to the Secretary of Education but the latter did not answer
to their letter.

On Oct. 31, 1990, students and their parents filed special civil actions for Mandamus, Certiorari and
prohibition, alleging that the respondents acted without or in excess of their jurisdiction and with grave
abuse of discretion in ordering their expulsion without prior notice and hearing, hence, in violation of their
right to due process, their right to free public education and their right to freedom of speech, religion and
worship. Petitioners prayed for the voiding of the order of expulsion or dropping from the rolls issued by
the District Supervisor; prohibiting and enjoining respondent from barring them from classes; and
compelling the respondent and all persons acting for him to admit and order their(Petitioners) re-
admission I their respective schools.

On November 27, 1990, Court issued a TRO and writ of preliminary mandatory injunction, commanding
the respondents to immediately re-admit the petitioners to their respective classes until further orders.

On May 31, the Solicitor General filed a consolidated comment to the petitions defending the expulsion
orders issued by the respondents.
TAMPUS, MARY GRACE G. CONSTITUTIONAL LAW 1

Petitioners stressed that while they do not take part in the compulsory flag ceremony, they do not engage
in external acts or behavior that would offend their countrymen who believe in expressing their love of
country through observance of the flag ceremony. They quietly stand at attention during the flag
ceremony to show their respect for the right of those who choose to participate in the solemn proceedings.
Since they do not engage in disruptive behavior, there is no warrant for their expulsion.

ISSUE:

Whether or not the expulsion of the members of Jehovahs Witness from the schools violates right receive
free education.

HELD:

Expulsion of the members of Jehovahs Witness from schools where they are enrolled violate their rights
as Philippine citizens, under the 1987 Constitution, to receive free education, for it is the duty of the state
to protect and promote the right of all citizens to quality education, and to make such education
accessible to all (Sec. I, Art XIV). Nevertheless, their right not to participate in the Flag Ceremony does
not give them a right to disrupt such patriotic exercises. If they quietly stand at attention during flag
ceremony while their classmates and teachers salute the flag, sing the national anthem and recite the
patriotic pledge, we do not see how such conduct may possibly disturb the peace, or pose a grave and
present danger of a serious evil to public safety, public morals, public health or any legitimate public
interest that the state has a right and duty to prevent.

It is appropriate to recall the Japanese occupation of our country in 1942-1944 when every Filipino,
regardless of religious persuasion, in fear of the invader, saluted the Japanese flag and bowed before
every Japanese soldier, perhaps if petitioners had lived through that dark period of our history, they would
not quibble now about saluting the Phil. Flag.

You might also like